RD Genitourinary formatted Flashcards

1
Q
1.	Middle aged male followed up for an incidental 3cm echogenic mass identified on ultrasound, the lesion is well demarcated and shows homogeneous high T1, low T2. 
A.	Simple cyst
B.	Renal cell carcinoma
c.	Haemorrhagic cyst
d.	Proteinaceous cyst
e.	Angiomyelolipoma
A

*LW: favoured answer of haemorrhagic cyst.

  • DDx for High T1 renal lesion = angiomyelolipoma, haemorrhagic cyst, proteinaceous cyst.
  • Promethius states DDx for Low T2 = papillary RCC, lipid poor AML and haemorrhagic cyst,
  • A lipid poor AML could account for a high T1 and low T2 appearances, but would have a heterogenous US appearance, not echogenic, being lipid poor.
  • Given haemorrhagic cyst would appear T1 and T2 bright with likely fluid debris levels, (follow up likely less than 72 hours), but can become T2 dark sub acutely.
  • Favoured answer is thus haemorrhagic cyst.
  • At MR imaging, papillary RCC frequently shows a pseudocapsule and frequently has low signal intensity on both T1- and T2-weighted images, whereas cRCC has higher signal intensity on T2-weighted images.

**LJS - favour haemorrhagic cyst.
Proteinaceous cyst can be high T1 but ?more likely remain high T2. Haemorrhagic cyst typically T1 high and T2 low (RP, Stat dx, Prometheus).

Prior notes:
b. Renal cell carcinoma ? T variable US appearance (50% hyperechoic, 40% isoechoic, 10% hypoechoic); variable MRI appearance but usually isointense on T1 & T2, but 40% are T1 hyperintense & most are T2 heterogenous. However papillary RCC is generally MR homogeneous, and is often high T1 & low T2I think this is a haemrrhagic cyst. 1. Middle aged male followed up for an incidental 3cm echogenic mass identified on ultrasound, the lesion is well demarcated and shows homogeneous high T1, low T2.

a. Simple cyst F anechoic, T1 low, T2 high
b. Renal cell carcinoma ? T variable US appearance (50% hyperechoic, 40% isoechoic, 10% hypoechoic); variable MRI appearance but usually isointense on T1 & T2, but 40% are T1 hyperintense & most are T2 heterogenous. However papillary RCC is generally MR homogeneous, and is often high T1 & low T2
c. Haemorrhagic cyst F on US contains internal echoes (acute) or thick walls with multiloculation (chronic); on MRI usually T1 hyperintense (subacute, < 72 hours) & T2 hyperintense (not as high as simple cyst) +/- fluid-debris level
d. Proteinaceous cyst ?T simulates haemorrhage (Statdx)
e. Angiomyolipoma F typically markedly echogenic on US; at MRI heterogenous with T1 hyperintense fat content, often T2 mod hyperintense; fat areas will decrease signal on fat sat or out of phase imaging; post Gd vascular portion will enhance, but fatty portion will not

AJR June 2009 On T2-weighted images, most papillary RCCs are hypointense and clear cell RCCs, hyperintense. The T2 hypointense appearance of papillary RCCs correlated with a predominant papillary architecture at pathology.
RG Nov 2006:Papillary RCCs typically appear hypovascular and homogeneous on imaging studies. Another important feature of papillary RCC is that bilateral and multifocal tumors are more common than in other subtypes of RCC (especially with hereditary syndromes) (11). Larger tumors show heterogeneity due to necrosis, hemorrhage, and calcification. Papillary RCC commonly demonstrates low signal intensity on T2-weighted MR images possibly due to the presence of by-products of hemorrhage and necrosis.

How well did you know this?
1
Not at all
2
3
4
5
Perfectly
2
Q
  1. Middle aged male followed up for an incidental 3cm echogenic mass identified on ultrasound, the lesion is well demarcated and shows homogeneous high T1, high T2.
    a. Simple cyst
    b. Renal cell carcinoma
    c. Haemorrhagic cyst
    d. Proteinaceous cyst
    e. Angiomyelolipoma
A

*LW: favoured answer of proteinaceous cyst.

**LJS - agree

  • DDx for High T1 renal lesion = angiomyelolipoma, haemorrhagic cyst, proteinaceous cyst.
  • A lipid poor AML could account for a high T1 and low T2 appearances, but would have a heterogenous US appearance, not echogenic, being lipid poor.
  • Given haemorrhagic cyst would appear T1 and T2 bright with likely fluid debris levels, (follow up likely less than 72 hours.
  • Favoured answer is thus proteinacesous cyst.
  • At MR imaging, papillary RCC frequently shows a pseudocapsule and frequently has low signal intensity on both T1- and T2-weighted images, whereas cRCC has higher signal intensity on T2-weighted images.
    RCC tends to occur in 50-70yr olds: unsure if this is considered older than middle age tho.

c. Haemorrhagic cyst T? on US contains internal echoes (acute) or thick walls with multiloculated (chronic); on MRI usually T1 hyperintense (subacute, < 72 hours) & T2 hyperintense (not as high as simple cyst) +/- fluid-debris leveld. Proteinaceous cyst T? simulates haemorrhage (Statdx)I think it’s more D2. Middle aged male followed up for an incidental 3cm echogenic mass identified on ultrasound, the lesion is well demarcated and shows homogeneous high T1, high T2. a. Simple cyst F anechoic, T1 low, T2 high
b. Renal cell carcinoma F variable US appearance (50% hyperechoic, 40% isoechoic, 10% hypoechoic); variable MRI appearance but usually isointense on T1 & T2, but 40% are T1 hyperintense & most are T2 heterogenous
c. Haemorrhagic cyst T? on US contains internal echoes (acute) or thick walls with multiloculated (chronic); on MRI usually T1 hyperintense (subacute, < 72 hours) & T2 hyperintense (not as high as simple cyst) +/- fluid-debris level
d. Proteinaceous cyst T? simulates haemorrhage (Statdx)
e. Angiomyelolipoma T typically markedly echogenic on US; at MRI heterogenous with T1 hyperintense fat content, often T2 mod hyperintense; fat areas will decrease signal on fat sat or out of phase imaging; post Gd vascular portion will enhance, but fatty portion will not.

How well did you know this?
1
Not at all
2
3
4
5
Perfectly
3
Q
  1. Young man with flank trauma, presents with hematuria. Has CT that shows large vascular heterogenous mass containing fat. Hemodynamically stable. What to do next?
    a. MRI
    b. DSA
    c. Family history
    d. Urgent surgery
A

b. DSA T best answer – look for active bleed which could be embolised at time; expect a highly vascular mass lesion with sacculated pseudoaneurysms & absence of AV shunts (AV shunting more in keeping with RCC)

Young man with flank trauma, presents with hematuria. Has CT that shows large vascular heterogenous mass containing fat. Hemodynamically stable. What to do next?

a. MRI ?F not sure how this would help – maybe of brain to look for TS??
b. DSA T best answer – look for active bleed which could be embolised at time; expect a highly vascular mass lesion with sacculated pseudoaneurysms & absence of AV shunts (AV shunting more in keeping with RCC)
c. Family history
d. Urgent surgeryPatients presenting with spontaneous bleeding are treated with embolization initially. Large tumours > 4cm should have intervention, either surgery or embolisation.

How well did you know this?
1
Not at all
2
3
4
5
Perfectly
4
Q

cause of renal vein thrombosis, which is false.

a. dehydration in kids.
b. amyloid
c. scleroderma
d. nephritis
e. nephritic syndrome

A

*LW: this is likely a bad recall, as Nephritic syndrome does not cause renal vein thrombosis, but presume the original question was nephrotic syndrome.
Scleroderma does NOT cause renal vein thrombosis, is an arterial process.
Therefore likely correct answer in this setting is scleroderma.

**LJS agree - all do except scleroderma and nephritic syndrome

E = F – RVT is assoc/ w/ nephrotic syndrome, not nephritic syndrome
A = T
B = T
C = ?T (can’t find a reference!)
D= if pyelonephritis T
E = F – RVT is assoc/ w/ nephrotic syndrome, not nephritic syndrome
How well did you know this?
1
Not at all
2
3
4
5
Perfectly
5
Q

Complication of post renal transplant. which is false?
a. reversal of flow indicates renal vein thrombosis
b. high RI in transplant renal artery is indicative of rejection
c. lymphoceles takes up something or
D. lymphoceles happens 3 or 4 days post op

A

D = lymphoceles seen typically 2-4 months post-op (StatDx). RG 2005 “They may develop at any time, from weeks to years after transplantation. However, they are usually an early complication, occurring within 1–2 months after transplantation.”

*LW:
Promethius states they lymphoceles are photopenic.
Nuk Medicine paper states “A lymphocele presents as a persistent photopenic area, although a mild degree of filling-in on delayed imaging has been observed”
Based on wording, would still agree 3-4 days post op is LEAST correct.

A – ? T this suggests RVT - although reversal of diastolic flow is non-specific (also seen in severe rejection & ATN), the combination of absent venous flow & reversed arterial diastolic flow is virtually diagnostic of RVT. Best diagnostic clue (StatDx) = “Echogenic material in renal vein with absence of flow on color Doppler US”

B = ? T – chronic rejection shows thin cortex & mild hydronephrosis – intrarenal RI may be normal or slightly raised with ↓ vascularity; acute rejection may have increased intrarenal RI, but not sensitive or specific. StatDx: rejection “not imaging diagnosis”.

C = no mention in StatDx of lymphocele taking up radiotracer; lymphoceles can extrinsically obstruct. RG 2005 “Radionuclide studies are helpful for excluding the presence of urine (urinoma)”.

D = lymphoceles seen typically 2-4 months post-op (StatDx). RG 2005 “They may develop at any time, from weeks to years after transplantation. However, they are usually an early complication, occurring within 1–2 months after transplantation.”Elevated RI (RI > 0.8 equivocal, RI > 0.9 more convincing) seen with severely increased vascular resistance in the kidney – from rejection or RVT.

How well did you know this?
1
Not at all
2
3
4
5
Perfectly
6
Q

regarding testicular U/S

a. microlithiasis a/w Kleinfelters
b. seminoma more cystic than embryonal
c. teratomas in kids less than 2 usually malignant

A

A = T = microlithiasis assoc/ w/ GCT, cryptorchidism, Klinefelter syndrome, Down syndrome, male pseudohermaphroditism

B = F = Seminomas are usually well-defined, hypoechoic, solid without calcification or tunica invasion; 1/3 of embryonal have cystic necrosis

C = F = In the child, differentiated mature teratomas usually follow a benign course. In the postpubertal male all teratomas are regarded as malignant, capable of metastatic behavior whether the elements are mature or immature (Robbins)

How well did you know this?
1
Not at all
2
3
4
5
Perfectly
7
Q
  1. Echogenic renal cystic structure on USS, high signal T1 and T2, no enhancement (VIC – shows enhancement), most correct:
    a. Haemorrhagic cyst
    b. Proteinaceous cyst
    C. Simple cyst
    d. AML
    e. ?
A

a. Haemorrhagic cyst T (but non enhancing)
b. Proteinaceous cyst F at US has ‘few scattered internal echoes’ (Statdx)
c. Simple cyst F not echogenic
d. AML F not cystic
e. ?

How well did you know this?
1
Not at all
2
3
4
5
Perfectly
8
Q
  1. Renal mass, high T1, low T2, diffuse marked enhancement, most correct
    a. Proteinaceous cyst
    b. Haemorrhagic cyst
    c. Simple cyst
    d. Papillary RCC
    e. Clear cell RCC
A

d. Papillary RCC ?T often T2 hypointense, show less enhancement (Hamm – Direct Diagnosis Urogenital imaging)
* LW: Promethius states papillary RCC included within T2 dark DDx (as well as lipid poor AML and haemorrhagic cyst), but also states are less vascular and not enhance equal to cortex on corticomedullary phase.

  1. Renal mass, high T1, low T2, diffuse marked enhancement, most correct
    a. Proteinaceous cyst F simulates haemorrhage (Statdx)
    b. Haemorrhagic cyst F on US contains internal echoes (acute) or thick walls with multiloculation (chronic); on MRI usually T1 hyperintense (subacute, < 72 hours) & T2 hyperintense (not as high as simple cyst) +/- fluid-debris level
    c. Simple cyst F anechoic, T1 low, T2 high
    d. Papillary RCC ?T often T2 hypointense, show less enhancement (Hamm – Direct Diagnosis Urogenital imaging)
    e. Clear cell RCC ?T often heterogenous (StatDx). Variable US appearance (50% hyperechoic, 40% isoechoic, 10% hypoechoic); variable MRI appearance but usually isointense on T1 & T2, but 40% are T1 hyperintense & most are T2 heterogenous. clear cell - high T1, high T2papillary - high t1, low t2RG

Rad-Path Papillary RCC May 2009At contrast material–enhanced computed tomography, pRCC enhances less than does cRCC in all phases of contrast-enhanced imaging. The difference in the degree of enhancement between pRCC and cRCC is due to differences in their intratumoral vascularity. In general, if a heterogeneous mass enhances to a degree similar to that manifested by the renal cortex, it is likely to be a cRCC. A mass that enhances to a lesser degree is likely to be a non–clear cell RCC (papillary or chromophobe RCC).AJR June 2009On T2-weighted images, most papillary RCCs are hypointense and clear cell RCCs, hyperintense. The T2 hypointense appearance of papillary RCCs correlated with a predominant papillary architecture at pathology.RG Nov 2006:Papillary RCCs typically appear hypovascular and homogeneous on imaging studies. Another important feature of papillary RCC is that bilateral and multifocal tumors are more common than in other subtypes of RCC (especially with hereditary syndromes) (11). Larger tumors show heterogeneity due to necrosis, hemorrhage, and calcification. Papillary RCC commonly demonstrates low signal intensity on T2-weighted MR images possibly due to the presence of by-products of hemorrhage and necrosis.

How well did you know this?
1
Not at all
2
3
4
5
Perfectly
9
Q
  1. Complications of renal transplantation
    a. Increased RI in the transplant artery is specific for rejection
    b. Lymphocoele accumulates tracer
    c. Lymphocoeles occur within 4/7 of operation
    d. Renal artery PSV of > 1m/s is diagnostic of stenosis
    e. Reversal of diastolic flow in renal artery suggests RVT
A

e. Reversal of diastolic flow in renal artery suggests RVT T – this suggests RVT - although reversal of diastolic flow is non-specific (also seen in severe rejection & ATN), the combination of absent venous flow & reversed arterial diastolic flow is virtually diagnostic of RVT
3. Complications of renal transplantation
a. Increased RI in the transplant artery is specific for rejection Fi. AR: Resistivity index (RI) lacks sensitivity and specificity, RI 0.8-0.9 suspicious, abnormal > 0.9ii. CR: RI may be normal or slightly raised with reduced vascularity

b. Lymphocoele accumulates tracer F
*LW: agree: promethius states they are photopenic.
Nuke med paper states: “A lymphocele presents as a persistent photopenic area, although a mild degree of filling-in on delayed imaging has been observed”
(https://journals.sagepub.com/doi/pdf/10.1177/2010105815611813)

c. Lymphocoeles occur within 4/7 of operation F lymphoceles seen typically 2-4 months post-op (StatDx)
d. Renal artery PSV of > 1m/s is diagnostic of stenosis F 1 m/s = 100cm/s - High peak systolic velocity waveform (200-250 cm/s) on Doppler examination indicates arterial stenosise. Reversal of diastolic flow in renal artery suggests RVT T – this suggests RVT - although reversal of diastolic flow is non-specific (also seen in severe rejection & ATN), the combination of absent venous flow & reversed arterial diastolic flow is virtually diagnostic of RVT

How well did you know this?
1
Not at all
2
3
4
5
Perfectly
10
Q
  1. MAG3 study with Captopril
    f. RAS
    g. PUJ obstruction
    h. VUR scan
    i. Assess for renal function
    j. RCC metastases
A

a. RAS T use ACE-I to evaluate renovascular HTN (Primer p940); can also use DTPA with captopril
4. MAG3 study with Captopril
a. RAS T use ACE-I to evaluate renovascular HTN (Primer p940); can also use DTPA with captopril
b. PUJ obstruction F use frusemide
c. VUR scan F usually use pertechnetate – alternatives are DTPA & sulphur colloid
d. Assess for renal functione. RCC metastases

How well did you know this?
1
Not at all
2
3
4
5
Perfectly
11
Q
  1. DMSA nuclear medicine renal scan is best for detecting (most correct):
    a. PUJ obstruction
    b. Scarring from reflux neprhopathy
    c. Metastatic RCC
    d. Renal hypertension
    e. Renal function
A

b. Scarring from reflux nephropathy
5. DMSA nuclear medicine renal scan is best for detecting (most correct):
i) PUJ obstruction F MAG3 (or DTPA) with frusemide
ii) Scarring from reflux nephropathy T
iii) Metastatic RCC
iv) Renal hypertension F DTPA or MAG3 with captoprilv) Renal function F DTPA best for GFR (100% glomerular filtration)

How well did you know this?
1
Not at all
2
3
4
5
Perfectly
12
Q
  1. DTPA nuclear medicine renal scan is best for detecting (most correct):
    a. PUJ obstruction
    b. Scarring from reflux nephropathy
    c. Metastatic RCC
    d. Renal hypertension
    e. Renal function
A

v) Renal function T DTPA is the agent of choice for quantification of GFR

How well did you know this?
1
Not at all
2
3
4
5
Perfectly
13
Q
  1. Regarding prostate cancer, which is most correct?
    a. More often arises in central zone
    b. More often hypoechoic than hyperechoic
    c. PSA is independent of gland size
    d. PSA normal in 50% of patients with prostate cancer
A

b. more often hypoechoic than hyperechoic T may be hypoechoic (60-70%), hyperechoic (1-5%), or isoechoic (30-40%) at TRUS [statdx]

  1. Regarding prostate cancer, which is most correct?
    a. more often arises in central zone F peripheral zone

b. more often hypoechoic than hyperechoic T may be hypoechoic (60-70%), hyperechoic (1-5%), or isoechoic (30-40%) at TRUS [statdx]
c. PSA is independent of gland size F In men without prostate cancer, serum PSA reflects the amount of glandular epithelium, which in turn reflects prostate size. Thus as prostate size increases with increasing age, the PSA concentration also rises; it increases at a faster rate in elderly men. [UTD]. PSA will go up with BPH, but goes up much higher per gram of tissue with cancer. If PSA is > 60, there are bony mets, and staging starts with a bone scan. If PSA is < 20, it is very uncommon to have bony mets.

d. PSA normal in 50% of patients with prostate cancer F NPV of 85% for a PSA value ≤4.0 ng/Ml [UTD]
* *LJS Robbins: 20-40% pt with organ confined prostate carcinoma has PSA < 4 (normal)

How well did you know this?
1
Not at all
2
3
4
5
Perfectly
14
Q

70 yo. testicular lesion.

a. seminoma.
b. lymphoma
c. yolk sac tumour

A

ANS = B (lymphoma) = Most common testicular tumour in men > 60, multiple lesions; 50% of cases bilateral (StatDx & Primer p337)

How well did you know this?
1
Not at all
2
3
4
5
Perfectly
15
Q

hypo echoic on renal u/s

a. aml
b. fungal ball
c. renal calculi
d. lymphoma.
e. ureteric stent

A

D = Lymphoma = T hypoechoic relative to renal parenchyma

A = AML = F echogenic

B = Fungal ball = F filling defect within collecting system

C = Renal calculi = F echogenic/shadowing

D = Lymphoma = T hypoechoic relative to renal parenchyma

How well did you know this?
1
Not at all
2
3
4
5
Perfectly
16
Q
  1. LEAST CORRECT regarding US imaging of renal cell carcinoma
    a. Approx 5% are isoechoic with renal parenchyma
    b. Majority of RCC under 3cm in size are more echogenic than renal parenchyma
    c. Demonstration of blood flow in substance of a complex cystic lesion is strong evidence of malignancy
    d. DDx of RCC includes focal parenchymal hypertrophy
    e. Approx 20% RCCs have detectable calcification
A

a. Approx 5% are isoechoic with renal parenchyma F hyper 48%, iso 42% and hypo 10% (statdx); often vascular; hypoechoic “pseudocapsule”
8. LEAST CORRECT regarding US imaging of renal cell carcinoma
a. Approx 5% are isoechoic with renal parenchyma F hyper 48%, iso 42% and hypo 10% (statdx); often vascular; hypoechoic “pseudocapsule”
b. Majority of RCC under 3cm in size are more echogenic than renal parenchyma T majority hyperechoic when smaller → a hyperechoic lesion at US needs CT evaluation to exclude RCC! (Zagoria, RANZCR ASM 2011)
c. Demonstration of blood flow in substance of a complex cystic lesion is strong evidence of malignancy T StatDx: Cyst vascularity greatly ↑ risk of malignancy
d. DDx of RCC includes focal parenchymal hypertrophy T column of Bertin
e. Approx 20% RCCs have detectable calcification T? up to 31% of RCCs contain calcium at CT (Zagoria p84); < 10% of RCC contain calcification (Statdx)Zagoria p84:• Up to 31% of RCCs contain calcium at CT• Peripheral calcification only: 80% are cysts• Central calcification & irregular: 87% are RCC’s• Mixed central & peripheral calcification: 50% are RCC’s, 50% are cysts.

How well did you know this?
1
Not at all
2
3
4
5
Perfectly
17
Q
  1. 30 year old male with lump on testes. On USS lump is well defined, 2cm, vascular and located in the epididymis. What is the most likely cause?
    a. Thromboses varix
    b. Adenomatoid tumour
    c. Epididymal cyst
    d. Rete appendix
    e. Lymphoma
A

2.Adenomatoid tumour - T - Slow growing mesothelial neoplasm, in 2nd to 4th decade, solid mass, can be up to 5cm in size, typically within epididymis (Dahnert), particularly the epididymal tail (Zagoria); 30% of extratesticular tumours (Chapman); most common epididymal tumour (Zagoria)

How well did you know this?
1
Not at all
2
3
4
5
Perfectly
18
Q
  1. Regarding the features of seminoma, which of the following is the most correct?
    a. Homogeneous on ultrasound
    b. Early infiltration into tunica vaginalis
    c. Bilateral
    d. Insensitive to radiation
    e. Common in children
A
  1. Homogeneous on ultrasound - T - tend to be homogeneously hypoechoic
  2. Regarding the features of seminoma, which of the following is the most correct? (JS)
  3. Homogeneous on ultrasound - T - tend to be homogeneously hypoechoic
  4. Early infiltration into tunica vaginalis - F - usually confined within the tunica albuginea
  5. Bilateral - F - bilateral in 1-3%, almost always asynchronous
  6. Insensitive to radiation - F - are radiosensitive versus NSGCT which are radioresistant 5.Common in children - F - average age is 40 years (Dahnert/Statdx)
How well did you know this?
1
Not at all
2
3
4
5
Perfectly
19
Q
  1. Which is false regarding horseshoe kidney?
    a. It increases the risk of RCC
    b. It often presents with UTI in children
    c. DMSA scan is routinely normal
    d. It is associated with obstruction
A

*LW:
Although StatDx states there is an increase in RCC, radiopedia, and literature review states no increase risk in RCC. Tumour incidence that is increased in horseshoe kidney includes: Wilms, TCC, and renal carcinoid. Donnelly 2nd Ed only mentions Wilms.
Commonly presents with UTI in children and is associated with obstruction.
Stat Dx doesn’t mention what specific study rather “Demonstrates fusion with functional parenchymal tissue
Can detect regional loss of function due to obstruction and inflammation”.
Quoted scarring in horseshoe kidneys is approx. 50%, and more than 50% have reflux, for which DMSA detects, thus it would appear that the DMSA scan would routinely be abnormal in horseshoe kidney.
Even more renal info….
–> Patients with hydronephrosis not due to VUR should be evaluated by a diuretic radionuclide scan with either (MAG-3 or technetium DTPA to differentiate hydronephrosis due to obstruction from nonobstructive urinary stasis
* thus appears both options A and C are false.

  1. DMSA scan is routinely normal F DMSA is used for investigation of horseshoe kidneys and can demonstrate amount of functional renal tissue
  2. Which is false regarding horseshoe kidney? (JS/SK)
  3. It increases the risk of RCC ? T Increased risk of malignancy – particularly Wilms (Primer) but with a small increased risk of adenocarcinoma (Donnelly). The incidence of renal cell cancer in the horseshoe kidney is no different from that of the normal kidney (eMedicine). StatDx says ↑ prevalence of RCC.
  4. It often presents with UTI in children T Associated with UTI in 30% (Primer)
  5. DMSA scan is routinely normal F DMSA is used for investigation of horseshoe kidneys and can demonstrate amount of functional renal tissueSK - ?? normal – StatDx – demonstrates fusion with functional parenchymal tissue. Can detect regional loss of function due to obstruction or inflammation. Considering the ↑ UTI, therefore would have possible cortical scarring at DMSA.
  6. It is associated with obstruction T Associated with PUJ obstruction in 30% (Primer)Emedicine:Certain cancers are more common in the horseshoe kidney.[1] This is thought to be due to teratogenic factors present at birth and the susceptibility of the diseased horseshoe kidney to certain cancers. Renal cell carcinoma is the most common renal cancer in horseshoe kidney, accounting for 45% of tumors.[2, 3] The incidence of renal cell cancer in the horseshoe kidney is no different from that of the normal kidney. Transitional cell cancer and sarcoma account for 20% and 7% of tumors, respectively. The relative risk of transitional cell carcinoma in the horseshoe kidney is increased 3- to 4-fold. This is thought to be due to chronic obstruction, stones, and/or infection in the affected kidneys. The incidence of both Wilms and carcinoid tumors is also higher in the horseshoe kidney. Examination of these tumors may provide an insight into the development and embryogenesis of the horseshoe kidney and the predilection of these two tumors to form in the horseshoe kidney.
How well did you know this?
1
Not at all
2
3
4
5
Perfectly
20
Q
  1. Vascular 2cm epididymal mass in 20 y.o. What is most likely?
    e. Torted hydatid of Morgagni
    f. Thrombosed varicocele
    g. Adenomatoid tumour
    h. Epidermoid
A
  1. Adenomatoid tumour - Slow growing mesothelial neoplasm, in 2nd to 4th decade, solid mass, can be up to 5cm in size, typically within epididymis (Dahnert), 30% of extratesticular tumours (Chapman); most common epididymal tumour (Zagoria)
  2. Vascular 2cm epididymal mass in 20 y.o. What is most likely? (JS)1.Torted hydatid of Morgagni = appendix testis, usually around 5mm, shouldn’t have flow if torted2.Thrombosed varicocele - Thrombosed varicocele shouldn’t have flow3.Adenomatoid tumour - Slow growing mesothelial neoplasm, in 2nd to 4th decade, solid mass, can be up to 5cm in size, typically within epididymis (Dahnert), 30% of extratesticular tumours (Chapman); most common epididymal tumour (Zagoria) 4.Epidermoid - Rare
How well did you know this?
1
Not at all
2
3
4
5
Perfectly
21
Q
  1. Which cystic renal mass is most likely to be malignant?
    i. A soft tissue component
    j. Thick, enhancing septa
    k. HU >20
    l. Rim calcification
A

1.A soft tissue component Bosniak 4 if there is enhancement (clearly malignant)

  1. Which cystic renal mass is most likely to be malignant? (JS)
  2. A soft tissue component Bosniak 4 if there is enhancement (clearly malignant)
  3. Thick, enhancing septa Bosniak 3 – surgical usually
  4. HU >20 Bosniak 2 - high density cyst considered benign if no enhancement post-contrast
  5. Rim calcification Bosniak 2 (low risk)

(Radiographics 2004 24:S101-115)

How well did you know this?
1
Not at all
2
3
4
5
Perfectly
22
Q
  1. Regarding seminoma, which of the following is true?
    a. Radiosensitive
    b. More aggressive than non seminomatous GCT
    c. Heterogenously hyperechoic on USS
    d. Stage II disease above diaphragm but no LN
A
  1. Radiosensitive Seminomas are radiosensitive versus NSGCT which are radioresistant (Robbins)
  2. Regarding seminoma, which of the following is true? (JS)
  3. Radiosensitive Seminomas are radiosensitive versus NSGCT which are radioresistant (Robbins)
  4. More aggressive than non seminomatous GCT NSGCT are more aggressive and have a poorer prognosis (Robbins)
  5. Heterogenously hyperechoic on USS Seminomas tend to be homogeneously hypoechoic (Primer)
  6. Stage II disease above diaphragm but no LN Stage II is distant spread, confined to retroperitoneal nodes below the diaphragm (Robbins)

Testicular staging:stage I: confined to testis, epididymis, spermatic cord, scrotumstage II: lymph nodes involved but no distant metastases and serum tumor markers are not very highstage III: distant metastases or moderately high serum tumor markers

How well did you know this?
1
Not at all
2
3
4
5
Perfectly
23
Q

1.Renovascular hypertension , which of the following are correct?

  1. Persistent dense nephrogram , delayed pyelogram with frusemide.
  2. Aorto:renal ratio of >2.5 means >70% stenosis.
  3. Fibromuscular dysplasia is cause in 50%.
  4. 15% of cause of hypertension.
A
  1. Persistent dense nephrogram , delayed pyelogram with frusemide. – T Pyelogram may be delayed if there is renal insufficiency
  2. Renovascular hypertension , which of the following are NOT incorrect? (GC/SK)
  3. Persistent dense nephrogram , delayed pyelogram with frusemide. – T Pyelogram may be delayed if there is renal insufficiency
  4. Aorto : renal ratio of >2.5 means >70% stenosis. - F >3.5 for >60% (Dahnert)
  5. Fibromuscular dysplasia is cause in 50%. - F 10-30%
  6. 15% of cause of hypertension. - F 1-5% of HTN (D)
How well did you know this?
1
Not at all
2
3
4
5
Perfectly
24
Q

2.Which is true in regards to renal trauma?

  1. Rim nephrogram on CT is seen in Vascular injury.
  2. Gross haematuria seen in all cases of severe trauma.
  3. Microscopic haematuria is an indication for CT.
  4. Lacerations communicate with pelvicalyceal system in majority.
A

1.Rim nephrogram on CT is seen in vascular injury. – T - devascularised kidney (grade 5), develops >8hrs post occlusion of main RA by intimal flap, reflects preserved perfusion by capsular a

a.Indications for CT:
Gross haematuria
Microhaematuria with shock
Microhaematuria with a positive DPL ( I assume a +ve FAST scan applies nowadays?)

How well did you know this?
1
Not at all
2
3
4
5
Perfectly
25
Q

3.Features of Xanthogranulomatous pyelonephritis?

  1. Kidney is typically small.
  2. 20% of cases have a renal calculus.
  3. Majority of cases are bilateral.
  4. Associated with pseudomonas infection in 70%.
  5. Diffuse involvement commoner than focal involvement.
A
  1. Diffuse involvement commoner than focal involvement. - T focal in 10-17%
  2. Features of Xanthogranulomatous pyelonephritis? (GC)
  3. Kidney is typically small. - F enlarged
  4. 20% of cases have a renal calculus. - F staghorn in 75%
  5. Majority of cases are bilateral. - F
  6. Associated with pseudomonas infection in 70%. - F proteus, e.coli
  7. Diffuse involvement commoner than focal involvement. - T focal in 10-17%
How well did you know this?
1
Not at all
2
3
4
5
Perfectly
26
Q

4.Features of Renal TB?

  1. Infundibulopapillary involvement is late.
  2. CXR is frequently abnormal
  3. Erosion of a papilla is a late feature.
  4. Papillary necrosis is seen.
A
  1. Papillary necrosis is seen. - T
  2. Features of Renal TB? (GC)
  3. Infundibulopapillary involvement is late. - F earliest sign is moth-eaten calyx (necrotizing papillitis)
  4. CXR is frequently abnormal - F normal in >50%
  5. Erosion of a papilla is a late feature. - F see (a)
  6. Papillary necrosis is seen. - TRef. Dahnert pg 984, Renal TB (RG 2004)
How well did you know this?
1
Not at all
2
3
4
5
Perfectly
27
Q

5.Concerning Prostate carcinoma?

  1. Hyperintense on T2 weighted images.
  2. Hyperechoic on US.
  3. < 10 PSA , bone involvement is unlikely.
  4. Rectal US is good for capsular spread.
A
  1. < 10 PSA , bone involvement is unlikely. - T Confined disease (stage A or B) in 75% with PSA <4, and 53% if 4-10
  2. Concerning Prostate carcinoma? (GC)
  3. Hyper intense on T2 weighted images. - F T2 hypointense focus within the normally hyperintense peripheral zone.
  4. Hyper echoic on US. - F most are hypo
  5. < 10 PSA , bone involvement is unlikely. - T Confined disease (stage A or B) in 75% with PSA <4, and 53% if 4-10
  6. Rectal US is good for capsular spread. - F – MRI for extracapsular extension (90% specific, 15% sensitive) – From ANZJ Surg 2007;77; 860-865 “The main role of TRUS in prostate cancer is for guiding biopsy. It has a positive predictive value (PPV) of 50–63% for the detection of ECE (BJU 2000). Moreover, overall detection is poor – up to 50% of nonpalpable tumours may not be visualized on grey-scale ultrasonography and only 17–57% of hypoechoic lesions in the peripheral zone are malignant”The reported accuracy of TRUS in the staging of PCa varies. Large multicentre study found an accuracy rate of only 62% However, a 1997 prospective multicentre study showed a continuously poor performance for TRUS, with its ability to detect ECE matching digital rectal examination (Ammersham)Ref. Dahnert pg 949
How well did you know this?
1
Not at all
2
3
4
5
Perfectly
28
Q
  1. Which is not a cause of renal vein thrombosis?
  2. Membranous glomerulonephritis.
  3. Gastroenteritis.
  4. Dehydration in infants.
  5. Scleroderma.
  6. Amyloid.
A
  1. Scleroderma. ? (StatDx lists PAN, SLE as causes)
  2. Which is not a cause of renal vein thrombosis ? (AB)
  3. Membranous glomerulonephritis.
  4. Gastroenteritis.
  5. Dehydration in infants.
  6. Scleroderma. ? (StatDx lists PAN, SLE as causes)
  7. Amyloid.

Renal vein thrombosis in infants most commonly due to dehydation/sepsis. In adults, most commonly due to nephrotic syndrome (which in turn is most commonly due to membranous glomerulonephritis). Amyloid is a recognised cause. Other important causes: RCC; trauma; post-transplantation; hypercoagulable states. Scleroderma affects the renal arterioles and causes cortical necrosis.Reference: Dahnert 6th ed.

How well did you know this?
1
Not at all
2
3
4
5
Perfectly
29
Q
  1. Least likely cause of increased renal medullary echogenicity?
  2. Medullary sponge kidney
  3. Hyperparathyroidism.
  4. Haemolytic uraemic syndrome.
  5. Sickle cell .
  6. RTA.
A

3.Haemolytic uraemic syndrome.

Medullary nephrocalcinosis occurs in medullary sponge kidney, hyperparathyroidism, renal tubular acidosis.
In sickle cell anaemia, renal echogenicity can be normal (most common) or mildly increased (medulla only or medulla and cortex).

HUS causes increased cortical echogenicity (from acute cortical necrosis).Reference: Dahnert 6th ed.; From the Archives of the AFIP: Sickle Cell Anaemia, Radiographics, 2001; Increased Renal Parenchymal Echogenicity: Causes in Pediatric Patients, Radiographics, 1990.

How well did you know this?
1
Not at all
2
3
4
5
Perfectly
30
Q
  1. Testicular imaging?
  2. Use 6MHZ probe.
  3. 3 calcific foci /field diagnostic of microlithiasis.
  4. Low resistance flow on Doppler is normal.
  5. Testicular cysts are always pathological
A

3.Low resistance flow on Doppler is normal.

A high-frequency linear transducer is used (7.5-10 MHz; higher in young children).

Low resistance flow is typical.

Microlithiasis is arbitrarily defined as 5 or more tiny (1-2 mm) hyperechoic and typically non-shadowing foci per image.

Reference: Gray-Scale and Color Doppler Sonography of Scrotal Disorders in Children: An Update, Radiographics, 2005

How well did you know this?
1
Not at all
2
3
4
5
Perfectly
31
Q
  1. Medial deviation of ureters T/F
  2. AP resection
  3. Prostatomegaly
  4. Ureterocele
  5. AAA
  6. Left retrocaval ureter
A
  1. AP resection - T
  2. Prostatomegaly - F – fish hook of distal ureters
  3. Ureterocele - F
  4. AAA – F *LW: FALSE - Promethius states this causes lateral displacement, as makes sense anatomically.
  5. Left retrocaval ureter - F – retrocaval ureter is RIGHT sided – which can cause medial deviation of the proximal ureter.

DDx (Dahnert)

Medial deviation of DISTAL ureter:
Hypertrophy of iliopsoas muscle
Enlargement of iliac lymph nodes
Aneurysmal dilatation of iliac vessels
Bladder diverticulum at VUJ (Hutch)
Following abdominoperineal surgery + retroperitoneal lymph node dissection
Pelvic lipomatosis
Renal cyst (parapelvic and lower pole)

Medial deviation PROXIMAL ureter:
Retrocaval ureter (right side only)
Retroperitoneal fibrosis
AAA (fibrosis if chronic leakage)(note: Primers puts prostatic enlargement as DDx for medial displacement- J-shaped—I am choosing to ignore this and consider fish hooking as separate to medial displacement)Added options ‘ d’ and ‘e’

How well did you know this?
1
Not at all
2
3
4
5
Perfectly
32
Q
  1. Medial deviation of ureters T/F
  2. AP resection
  3. Prostatomegaly
  4. Ureterocele
  5. AAA
  6. Left retrocaval ureter
A
  1. AP resection - T
  2. Prostatomegaly - F – fish hook of distal ureters
  3. Ureterocele - F
  4. AAA – FALSE
  5. Left retrocaval ureter - F – retrocaval ureter is right sided – which can cause medial deviation of the proximal ureter.
DDx (Dahnert)Medial deviation of DISTAL ureter:
Hypertrophy of iliopsoas muscle
'Enlargement of iliac lymph nodes
'Aneurysmal dilatation of iliac vessels
Bladder diverticulum at VUJ (Hutch)
Following abdominoperineal surgery + retroperitoneal lymph node dissection
Pelvic lipomatosis
Renal cyst (parapelvic and lower pole)
Medial deviation PROXIMAL ureter:
Retrocaval ureter (right side only)
Retroperitoneal fibrosis
AAA (fibrosis if chronic leakage)
(note: Primers puts prostatic enlargement as DDx for medial displacement- J-shaped—I am choosing to ignore this and consider fish hooking as separate to medial displacement)

Added options ‘ d’ and ‘e’

How well did you know this?
1
Not at all
2
3
4
5
Perfectly
33
Q

12.Regarding HIV related renal problems

  1. Nelfinavir related calculi cannot be seen on CT
  2. Indinavir related calculi are radiolucent
  3. Majority of HIV associated nephropathy kidneys have enlarged kidneys
  4. Only a minority of HIV-infected patients develop renal impairment during their illness.
  5. Hypertensive nephropathy is the leading cause of renal failure in HIV patients
A
  1. Indinavir related calculi are radiolucent - T – (true for pure stones, however can be detected on CT if contain other substances – Ca++, uric acid)
  2. Regarding HIV related renal problems (TW)
  3. Nelfinavir related calculi cannot be seen on CT – F – unlike indinivar – nelfinavir stones have been reported to be seen on CT, pure indinivar stones not seen on all modalities.
  4. Indinavir related calculi are radiolucent - T – (true for pure stones, however can be detected on CT if contain other substances – Ca++, uric acid)
  5. Majority of HIV associated nephropathy kidneys have enlarged kidneys – F – majority have normal-sized kidneys (Radiographics 2008)
    * LW: TRUE as Radiopedia disagrees with above - longitudinal renal size is increases, with increased parenchymal echogenicity, and decreased renal sinus fat. CT also shows enlarged kidneys. Diagnosis is biopsy proven.
  6. Only a minority of HIV-infected patients develop renal impairment during their illess. – F – most HIV infected patients develop RI. (Radiographics 2008)
  7. Hypertensive nephropathy is the leading cause of renal failure in HIV patients – F – HIVAN is leading cause (40% of cases of HIV related renal disease). (Radiographics 2008)Indinovir (protease inhibitor for treatment of HIV-type 1) – precipitations of drug crystals in renal tubule.
How well did you know this?
1
Not at all
2
3
4
5
Perfectly
34
Q
  1. Medial deviation of ureters in the bony pelvis is seen T/F
  2. In pelvic adenopathy
  3. In prostatomegaly
  4. In herniation of urether through sacro-sciatic foramen
  5. In ureterocele
  6. Following abdomino-peritoneal resection
A

19.Medial deviation of the ureter in the bony pelvis is seen T/F (TW)

  1. In pelvic adenopathy - T
  2. In prostatomegaly - F - fish hook of distal ureters
  3. In herniation of the ureter through the sacro-sciatic foramen - F
  4. In ureterocele - F
  5. Following abdomino-peritoneal resection – T
How well did you know this?
1
Not at all
2
3
4
5
Perfectly
35
Q
  1. What is not a cause of increased RI
  2. acute pyelonephritis
  3. ATN
  4. Ureteric obstruction
  5. Hepatorenal syndrome
  6. Diabetes
A

5.Diabetes ?? F - occurs late - however more there is literature saying can actually occur early with DM too.

AJR 2003 (The resistive Index in Renal Doppler Sonography: Where Do We Stand?)Doppler RI = ([PSV – EDV] / PSV) = 1-(EDV/PSV)Most consider RI 0.70 to be upper threshold of normal in adults (with exceptions: commonly >0.70 in 1st year of life, and can be >0.70 up to 4yo).Elevation of RI more likely with vascular / tubulointerstitial process, less likely with glomerular disease. Elevation of RI:Vascular:Acute vascular rejectionRenal vein thrombosisincreased intraparencyhmal pressure :ATN: elevated resistive index >0.75 (in 91% ) – where as pre-renal ARF cause has normal RI. RI greater than 0.07 was found to be a reliable discriminator between acute tubular necrosis and prerenal failure (AJR, Dahnert)Pyelonephritisperinephric / subcapsular fluid collectionrenal transplants (ATN / rejection)Cardiacsignificant systemic hypotensionmarkedly decreased heart rate (converse is true as well)Diabetes (?late finding) – combination of small vessel disease plus abnormal cardiac function in T2DM patients – generalized reduction in vascular compliance associated with arteriosclerosis in a range of vascular beds. in neonates + infants: normal findingHRS – develop renal dysfunction in pts with acute or chronic, severe liver disease - ?due to reduced renal vlood flow and vasoconstriction (due to local & systemic factors).Ureteric obstruction: > 0.7 - Obstructive cause initially thought to be mechanical cause (ie pressure increase) – but research suggests complex interactions between several regulatory pathways (RAS, kallikrein-kinin, PG-thromboxane) are responsible for intense, post obstructive renal vasoconstriction.

How well did you know this?
1
Not at all
2
3
4
5
Perfectly
36
Q

15.Which of the following statements about Phaeochromocytoma is false?

  1. 10% are bilateral/multiple, 10% are extradrenal and 10% are malignant
  2. Hemihypertrophy is a known association
  3. contains central heterogenous areas of decreased signal intensity in 35% on T2WI
  4. I-131 MIBG is used to treat metastases
  5. does not contain fat
A

2.Hemihypertrophy is a known association - F Phaeo is the 10 % TUMOUR 10% are bilateral/multiple, 10% are extradrenal and 10% are malignant and 10% are familial and 10% have speckled calcification

Hemihypertrophy is assoc with Wilms tumor.

Phaeo associations:
MEN II
Neuroectodermal disorders - NF, VHL, TS Familial Phaeo

I think this is meant to be Carney’s Triad = Gastric epithelioid leiomyosarcoma, functioning exra-adrenal paraganglioma, pulmonary chondromas) rather than Carney syndrome (Cardiac Myxoma, hyperpigmented skin lesions, breast fibroadenoma, psammomatous melanotic schwannoma, pituitary adenoma, large cell calcifying sertoli cell tumour, primary pigmented nodular adrenocortical hyperplasia)

How well did you know this?
1
Not at all
2
3
4
5
Perfectly
37
Q
  1. In the pediatric patient, which is the least likely cause of echogenic medulla?
  2. RTA
  3. Sickle cell
  4. HUS
  5. MSK
  6. HPT
A
  1. HUS - F - Haemolytic Uraemic Syndrome: triad of thrombocytopaenia, microangiopathic haemolytic anaemia and acute oliguric renal failure leading to uraemia. HUS leads to acute cortical necrosis and cortical nephrocalcinosis [prerequisites US]
  2. In the pediatric patient, which is the least likely cause of echogenic medulla? (CC)
  3. RTA - T RTA renal parenchymal disease is often a pathophysiological type diagnosis with histology and renal function parameters including Renal tubular acidosis (RTA) which leads to chronic metabolic alkalosis as the distal nephrons are unable to resorb bicarbonate or secrete hydrogen (depending on the type I-IV). [GU pre requisites and Merck Manual]
  4. Sickle cell -T Sickle Cell Disease – rare cause. Can cause cortical (eMed) and medullary nephrocalcinosis (Dahnert 6th pg 894).
  5. HUS - F - Haemolytic Uraemic Syndrome: triad of thrombocytopaenia, microangiopathic haemolytic anaemia and acute oliguric renal failure leading to uraemia. HUS leads to acute cortical necrosis and cortical nephrocalcinosis [prerequisites US]
  6. MSK - T Medullary Sponge Kidney = benign tubular ectasia. Leads onto medullary nephrocalcinosis in 40-80%. Recognised associations [Dahnert]:Parathyroid adenoma, Caroli’s Disease, Ehlers Danlos Syn
  7. HPT - T All causes of hypercalcaemia can result in echogenic medulla.
How well did you know this?
1
Not at all
2
3
4
5
Perfectly
38
Q
  1. Regarding renal vein thrombosis, which of the following is not a risk factor? (CC)
  2. dehydration
  3. membranous GN
  4. amyloid
  5. scleroderma
  6. nephrotic syndrome
A

4.scleroderma - F - SLE does

How well did you know this?
1
Not at all
2
3
4
5
Perfectly
39
Q
  1. Which of the following is not a cause of bilateral renal enlargement?
  2. TS
  3. Leukemia
  4. RTA
  5. Inf mononucleosis
  6. HUS
A

4.Inf mononucleosis F
Inf mononucleosis –T. Usually associated with splenomegaly and Epstein Barr virus (EBV) = a tetrad of fever, pharyngitis, fatigue and lymphadenopathy. However, EBV genome is found is a minority of Burkitt Lymphoma which can enlarge the kidneys. I think this is too much of a stretch for this question hence True - not a cause of bilateral renal enlargement [Merck, and Dahnert]

  1. Which of the following is not a cause of bilateral renal enlargement? (CC)
  2. TS T TS : multiple cysts are found in cortex and medulla in 10 – 15% mimicking adult polycystic kidney disease
  3. Leukemia T Leukaemia is a recognized cause of smooth bilateral renal enlargment from infiltrative replacement of renal interstitial tissue.
  4. RTA T RTA is a functional deficit, which can later lead to chronic renal failure from nephrocalcinosis (chronic interstitial nephritis with cellular infiltration).
  5. Inf mononucleosis FInf mononucleosis –T. Usually associated with splenomegaly and Epstein Barr virus (EBV) = a tetrad of fever, pharyngitis, fatigue and lymphadenopathy. However, EBV genome is found is a minority of Burkitt Lymphoma which can enlarge the kidneys. I think this is too much of a stretch for this question hence True - not a cause of bilateral renal enlargement [Merck, and Dahnert]
  6. HUS T - normal to slightly inc in size
How well did you know this?
1
Not at all
2
3
4
5
Perfectly
40
Q

19.Which of the following statements about duplex renal systems is incorrect?

  1. ectopic insertion gives bed wetting in boys
  2. PUJ obstruction of lower system
  3. Retention is a presenting symptom in both sexes
  4. Reflux lower system
  5. Epididymo orchitis is a presenting symptom
A
  1. ectopic insertion gives bed wetting in boys F – boys always have ectopic insert above sphincter therefore no enuresis
  2. Which of the following statements about duplex renal systems is incorrect? (CC)
  3. ectopic insertion gives bed wetting in boys F – boys always have ectopic insert above sphincter therefore no enuresis
  4. PUJ obstruction of lower system T – usually involves lower pole (LJS edit: combined PUJ + duplex collecting system is case report level rare)
  5. Retention is a presenting symptom in both sexes T
  6. Reflux lower system T
  7. Epididymo orchitis is a presenting symptom T – in preadolescent malemales do not have infrasphincteric insertion, only females [Dahnert]PUJ obstruction from extrinsic vessels usually effects the lower pole (anterior in 25-39%, posterior in 5-10%) )selectively involving the lower system. Presenting symptoms of duplex systems: [Weiss, comprehensive urology]UTI – most common Loin pain but also central and epigastric that may be associated with vomitingNeonatal abdominal mass – most common cause of neonatal hydronephrosisHaematuriaHypertension VUR affects the lower moiety more commonly due to a shorter submucosal tunnel [Dahnert]Although rare, ectopic insertion can be in the seminal vesicles (because of relationship with mesonephros) and presents with epididymo-orchitis or UTI rather than urinary incontinence.
How well did you know this?
1
Not at all
2
3
4
5
Perfectly
41
Q
  1. Regarding neonatal adrenal haemorrhage, which is false:
  2. It is usually bilateral
  3. More common on the right
  4. More common with a breech presentation
  5. Occurs in the first 7 days
  6. ?Increased with respiratory distress
A
  1. It is usually bilateral F - 10% bilateral
  2. Regarding neonatal adrenal haemorrhage, which is false: (TW)
  3. It is usually bilateral F - 10% bilateral
  4. More common on the right T – R>L 7:3
  5. More common with a breech presentation T – difficult labor / delivery / breech
  6. Occurs in the first 7 days T – first week of life
  7. Increased with respiratory distress T – assoc with hypoxia /neonatal stress

Aetiol : Neonatal stressDifficult labour – forceps, breechHypoxia/asphyxia due to prematuritySepsisHaemorrhagic disorders eg.DICIncreased risk LGA, diabetic mums. (Dahnert 6th pg 918)

How well did you know this?
1
Not at all
2
3
4
5
Perfectly
42
Q

21.Prostate cancer, which is false:

  1. More common in central zone
  2. Can be hyperechoic on ultrasound
  3. Transrectal ultrasound poor for evaluating capsular invasion
  4. Bone metastasis not common if asymptomatic and PSA <10
A
  1. More common in central zone F – Periph zone 80%, TZ 15% CZ 5%. Multifocal in 40% (Dahnert 6th)
  2. Prostate cancer, which is false: (TW)
  3. More common in central zone F – Periph zone 80%, TZ 15% CZ 5%. Multifocal in 40% (Dahnert 6th)
  4. Can be hyperechoic on ultrasound T – can be but nb. 61% hypoechoic, mixed 2%, hyperechoic 0nly 2% (Dahnert 6th).
  5. Transrectal ultrasound poor for evaluating capsular invasion T – see below
  6. Bone metastasis not common if asymptomatic and PSA <10 T - Disease confined to gland 53% pt with PSA 4-10mg/mL (Dahnert 6th)

From ANZJ Surg 2007;77; 860-865 “The main role of TRUS in prostate cancer is for guiding biopsy. It has a positive predictive value (PPV) of 50–63% for the detection of ECE (BJU 2000). Moreover, overall detection is poor – up to 50% of nonpalpable tumours may not be visualized on grey-scale ultrasonography and only 17–57% of hypoechoic lesions in the peripheral zone are malignant”The reported accuracy of TRUS in the staging of PCa varies. Large multicentre study found an accuracy rate of only 62% However, a 1997 prospective multicentre study showed a continuously poor performance for TRUS, with its ability to detect ECE matching digital rectal examination (Ammersham)

How well did you know this?
1
Not at all
2
3
4
5
Perfectly
43
Q

22.Renovascular hypertension, which is false:

  1. Systolic – diastolic / systolic equals index
  2. Aortorenal ratio of 2.5 means >than 70% stenosis
  3. Hyperpulsatility means a more distant stenosis
  4. 0.12 sec acceleration time intrarenal arteries indicates proximal stenosis
A
  1. Aortorenal ratio of 2.5 means >than 70% stenosis F - > 3.5 for >70%
  2. Renovascular hypertension, which is false: (TW)
  3. Systolic – diastolic / systolic equals index T
  4. Aortorenal ratio of 2.5 means >than 70% stenosis F - > 3.5 for >70%
  5. Hyperpulsatility means a more distant stenosis - T - before a stenosis will have high resistance wave form - ie little, no, or reversed diastolic flow with high arterial flow (trying to get through stenosis). At stenosis have turbulent flow, and distal to stenosis the arterial flow drops and have a low resistance wave form - ie tardus parvus.
  6. 0.12 sec acceleration time intrarenal arteries indicates proximal stenosis - T AT > 0.07s (indirect sign)
44
Q

23.Regarding fibromuscular dysplasia of the renal artery:

  1. Most common at the ostium
  2. Recurs post dilation
  3. Osteal stenosis
  4. More commonly in the elderly
A
  1. Recurs post dilation T - but PTA 90% success rate with very low restenosis rate (Dahnert)
  2. Regarding fibromuscular dysplasia of the renal artery: (TW)
  3. Most common at the ostium F – 1st 1/3 of the renal artery spared in 98%
  4. Recurs post dilation T - but PTA 90% success rate with very low restenosis rate (Dahnert)
  5. Osteal stenosis F – see (a)
  6. More commonly in the elderly F – most common cause of renovascular hypertension in children and young adults (<30-40y).FMD – involves mid and distal renal artery 79%. (Dahnert 6th)
45
Q
  1. Testicular ultrasound:
  2. Can tell teratoma from seminoma
  3. 10% have cysts
  4. TB only effects epididymitis
A
  1. 10% have cysts T – 8-10% males (RG 2002).
  2. Testicular ultrasound: (TW)
  3. Can tell teratoma from seminoma F – although age group, and some sonosgraphic appearances may help (see below), probably cannot differentiate confidently. “With rare exception, solid intratesticular masses should be considered malignant (Radiographics 2002)”.
  4. 10% have cysts T – 8-10% males (RG 2002).
  5. TB only affects the epididymus F – seminoma – uniformly hypoechoic. Larger tumors more heterogeneous. Cystic component 10%. Avg age is approx 40yo.teratoma – well-circumscribed complex masses. Cysts common feature. Heterogeneous. Echogenic foci. Children – 75% occur <5yo.Benign testicular cysts – either in tunica albuginea or parenchyma. Tunica albuginea cysts – peripherally located and may be single or multiple. Intratesticular cysts usually near mediastinum testis. If any solid component need to treat as malignant (and DDx from cystic lesions – Teratoma).Tuberculous epididymitis from ascending/descending route of infection. Tuberculous orchitis usually from direct extension from epididymal infection, rarely from hematogenous spread.
46
Q

25.Testicular torsion:

  1. Increased flow post spontaneous untorsion recognized
  2. Epididymis and testes involved vs just testical inflammation vs torsion
  3. Normal grey-scal appearance of the testis excludes torsion
  4. Colour duplex imaging has sensitivity and specificity of ~60%
A
  1. Increased flow post spontaneous untorsion recognized - T – normal or increased flow may be seen with spontaneous detorsion (eMed 2008, Dahnert 6th pg975)
  2. Testicular torsion: (TW)
  3. Increased flow post spontaneous untorsion recognized - T – normal or increased flow may be seen with spontaneous detorsion (eMed 2008, Dahnert 6th pg975)
  4. Epididymis and testes involved vs just testical inflammation vs torsion - ?not sure what this stem is
  5. Normal grey-scale appearance of the testicles excludes torsion - F – can have normal appearance in first 6hrs.
  6. Colour duplex imaging has sensitivity and specificity of ~60% - F – 86% sensitive, 100% specific, 97% accurate (Dahnert 6th)
47
Q
  1. Which of the following is not a cause for a small renal artery aneurysm?
  2. Diabetes
  3. Transplant rejection
  4. IVDU
  5. SLE
  6. RCC
A

1.Diabetes F

Renal artery aneuyrsms – 2/3rd extrarenal, 1/3rd intrarenal

Extrarenal aneuyrsms
Atherosclerotic (most common), FMD, pregnancy, NF, Ehlers Danlos Trauma, Behcet, mycotic (staphy, strep, pneumococcus, salmonella)

Intrarenal Aneurysm
Congenital Arteritis: Incl PAN, wegeners, Tx rejection, drug abuse, SLE Degenerative- Atherosclerosis
Tumour: incl RCC, AML
Mesenchymal disease: NFTraumaInfection: Syphilis, TB (Dahnert 6th pg 894)

48
Q

27.Regarding adrenals, which statement is true?

  1. Calcification excludes malignancy
  2. Adrenal hyperplasia is less common in malignancy
  3. 5% Cushings associated with adrenal carcinoma
  4. Conn’s verses Cushings can be differentiated on CT
A
  1. 5% Cushings associated with adrenal carcinoma T
  2. Regarding adrenals, which statement is true? (TW)
  3. Calcification excludes malignancy F – Adrenocortical adenocaricinoma – calcified 30%, Phaeo Ca+ in 10%
  4. Adrenal hyperplasia is less common in malignancy F
  5. 5% Cushings associated with adrenal carcinoma T
  6. Conn’s verses Cushings can be differentiated on CT F

Adrenal Ca+ (Chapman and Nak, Dahnert 6th)child: cystic disease (usually from haemorrhage), neuroblastoma, ganglioneuroma, Wolman’s disease.

Adult: cystic disease, carcinoma, addison’s, ganglioneuroma, inflammatory, phaeo.Adrenocortical carcinoma – large size and calcification suggests malignancy (D 6th)

Adrenocortical hyperplasia has 4x increase in incidence in patients with malignancy (D 6th)Cushings syndrome assoc with adrenocortical carcinoma 5-10% cases (10% in adults, 66% children) (D 6th)

49
Q
  1. Renal trauma, which is false:
  2. Large subcapsular haematomas may appear biconvex
  3. Pelvic kidney is at risk of trauma
  4. A rim nephrogram may not be seen in the acute setting of traumatic renal infarction
  5. Grade 3 lacerations do not involve the collecting system
  6. Delayed scanning is always indicated in cases of suspected renal trauma
A
  1. Delayed scanning is always indicated in cases of suspected renal trauma – F – indicated if the initial CT shows a deep parenchymal laceration or a large perirenal fluid collection.
  2. Renal trauma, which is false: (GC)
  3. Large subcapsular haematomas may appear biconvex - T – small ones are Crescentic, large exert mass effect and appear biconvex
  4. Pelvic kidney is at risk of trauma - T
  5. A rim nephrogram may not be seen in the acute setting of traumatic renal infarction – T – indicates a devascularised kidney (grade 5), develops >8hr post occlusion of main RA by intimal flap, reflects preserved perfusion by capsular aa.
  6. Grade 3 lacerations do not involve the collecting system - T – Grade 3: lacerations >1cm deep without extension into collecting system or urinary extravasation
  7. Delayed scanning is always indicated in cases of suspected renal trauma – F – indicated if the initial CT shows a deep parenchymal laceration or a large perirenal fluid collection.Ref. Dahnert pg 981, CT Blunt Renal Trauma (RG 2001)
50
Q

29.Fetal genitourinary anomalies, which is false:

  1. Primary megaureter is due to an alteration at the VUJ
  2. Bilateral renal agenesis results in oligohydramnios
  3. PUJ is bilateral in 70%
  4. Hydrops due to a renal cause leads to a bad outcome
  5. Urogenital sinus malformations occur only in phenotypic girls
A
  1. PUJ is bilateral in 70% F - bilateral in 10-40%
  2. Fetal genitourinary anomalies, which is false: (GC)
  3. Primary megaureter is due to an alteration at the VUJ T - 1° megaureter is due to an idiopathic congenital alteration at the VUJ (obstructed/aperistaltic, refluxing, or non-refluxing/unobstructive). 2° megaureter is due to bladder or urethral obstruction (eg. PUV).
  4. Bilateral renal agenesis results in oligohydramnios T - Potter sequence: pulmonary hypoplasia, bowing of legs, loose skin, growth retardation. Other renal causes include PUV, ARPKD, MCDK
  5. PUJ is bilateral in 70% F - bilateral in 10-40%
  6. Hydrops due to a renal cause leads to a bad outcome T – high mortality
  7. Urogenital sinus malformations occur only in phenotypic girls T – like cloacal malformation
51
Q

30.There is a recognised association between all of the following EXCEPT:

  1. Pelvicalyceal obstruction and Xanthogranulomatous pyelonephritis
  2. Calcification in a non-peripheral renal mass and malignancy
  3. Medullary sponge kidney and Caroli’s disease
  4. Wolman’s disease and adrenal atrophy
  5. Schistosomiasis and bladder calcification
A
  1. Wolman’s disease and adrenal atrophy = Primary familial xanthomatosis. AR inheritance, deficiency of lysosomal acid esterase. Bilateral punctate calcification throughout adrenals, but maintain their normal shape. Also hepatosplenomegaly, SB wall thickening, generalized osteoporosis.
  2. There is a recognised association between all of the following EXCEPT: (GC)
  3. Pelvicalyceal obstruction and Xanthogranulomatous pyelonephritis T - 75% have staghorn calculus.
  4. Calcification in a non-peripheral renal mass and malignancy T - Calcification within a solid renal mass is indicative of malignancy, whereas in a cystic renal mass, calcification can be seen in the wall/septa of benign or malignant lesions.*
  5. Medullary sponge kidney and Caroli’s disease T - 80% of Caroli’s have medullary sponge kidney.
  6. Wolman’s disease and adrenal atrophy = Primary familial xanthomatosis. AR inheritance, deficiency of lysosomal acid esterase. Bilateral punctate calcification throughout adrenals, but maintain their normal shape. Also hepatosplenomegaly, SB wall thickening, generalized osteoporosis.
  7. Schistosomiasis and bladder calcification T - S. haematobium. Bladder wall calcifn in up to 56% - begins at base and involves all wall layers. End stage shows reduced bladder capacity + post void residual. May also see bladder calculi, filling defects, distal ureteral calcifn or strictures.
52
Q

31.Which is not a cause of renal vein thrombosis

  1. Membranous glomerulonephritis
  2. scleroderma
  3. gastroenteritis
  4. Dehydration in infants
  5. Amyloid
A
  1. Scleroderma F – SLE does.
  2. Which is not a cause of renal vein thrombosis (GC)
  3. Membranous glomerulonephritis T – Nephrotic syndrome is a hypercoagulable state – heavy proteinuria leads to decreased antithrombin III, relative excess of fibrinogen, and other changes in clotting factors.
  4. Scleroderma F – SLE does.
  5. gastroenteritis T – secondary to dehydration
  6. Dehydration in infants T
  7. Amyloid T - Accumulation of eosinophilic protein substances
53
Q
  1. Complications of renal transplantation include all except?:
  2. Avascular necrosis
  3. Amyloid
  4. Hypoparathyroidism
  5. An increased incidence of lymphoma
A
  1. Hypoparathyroidism F
  2. Complications of renal transplantation include all except?: (GC)
  3. Avascular necrosis T - Most commonly occurs in femoral head, bilateral in 60-80%. May be due to steroids, secondary to hyperparathyroidism, liver disease, dialysis (incr protein catabolism
  4. Amyloid T – Cx of CRF
  5. Sift tissue calcification around the joints T – from secondary hyperparathyroidism
  6. Hypoparathyroidism F
  7. An increased incidence of lymphoma T – Prevalence of NHL is 35x greater than in general population. Also increased risk of skin cancers and RCC
54
Q

33.Prostate carcinoma, which is true:

  1. Prostatic carcinoma arises more often in the central zone than in the peripheral zone
  2. Prostatic carcinomas are more often hypoechoic than hyperechoic compared with adjacent prostate
  3. The serum PSA level, as a predictor of prostatic carcinoma, is independent of prostate size
  4. About 50% of patients with prostatic carcinoma have a normal serum PSA level
  5. Haematospermia is due to prostatic carcinoma in about 75% of cases
A
  1. Prostatic carcinomas are more often hypoechoic than hyperechoic compared with adjacent prostate
  2. Prostate carcinoma, which is true: (GC)
  3. Prostatic carcinoma arises more often in the central zone than in the peripheral zone - F 80% in periph zone
  4. Prostatic carcinomas are more often hypoechoic than hyperechoic compared with adjacent prostate
  5. The serum PSA level, as a predictor of prostatic carcinoma, is independent of prostate size - F
  6. About 50% of patients with prostatic carcinoma have a normal serum PSA level - F 19-30%
  7. Haematospermia is due to prostatic carcinoma in about 75% of cases - F rare cause CME 02.38 Ref. Dahnert pg 949
55
Q

34.With regards to bladder cancer, which is true?

  1. TCC hypervascular on angio
  2. CT is better than surgery for extravesical spread
  3. Internal iliac LN first involved
  4. CT is good for detecting seminal vesicle invasion by losing fat plane
A
  1. CT is better than surgery for extravesical spread ?T – see below. Obviously Sx will not pick up a lung met. However is ?Sx better for local extension.
  2. With regards to bladder cancer, which is true? (TW)
  3. TCC hypervascular on angio ??? with fast dynamic contrast-enhanced imaging, bladder cancer enhances more avidly and earlier than other tissues in the normal bladder. But bladder Ca is heterogeneous – so may have variable enhancement?
  4. CT is better than surgery for extravesical spread ?T – see below. Obviously Sx will not pick up a lung met. However is ?Sx better for local extension.
  5. Internal iliac LN first involved F – various lymphatic drainage – superior bladder to ext iliac, inferior to int iliac, some around neck region to sacral or common iliac nodes.
  6. CT is good for detecting seminal vesicle invasion by losing fat plane F – loss of fat plane may relate to inflammation / tumor involvement

Bladder carcinoma Radiographics Article 2006.

CT 52% accurate due to ‘overstaging’
“imaging is important for accurate staging … detection of pelvic side wall invasion or lymph adenopathy as clinical staging is inaccurate”“sensitivity and specificity for detecting perivesical spread with 92% and 98% respectively and accuracy of 96%”.

56
Q

35.On CT of the adrenals:

  1. The right gland characteristically occupies a more caudal position to the left.
  2. Conn’s adenomata are characteristically less than 3cm in diameter.
  3. Calcification within a mass indicates that it is benign.
  4. Fat within a mass indicates an angiomyolipoma.
  5. Both glands are normal in a patient with Cushings disease
A
  1. Conn’s adenomata are characteristically less than 3cm in diameter. T – aldosteronoma – 1.7cm avg size.
  2. On CT of the adrenals: (TW)
  3. The right gland characteristically occupies a more caudal position to the left. F – eMed, however I’ve found the occasional contradicting answer.
  4. Conn’s adenomata are characteristically less than 3cm in diameter. T – aldosteronoma – 1.7cm avg size.
  5. Calcification within a mass indicates that it is benign. F – Adrenocortical carcinoma, phaeo, cysts can all calcify.
  6. Fat within a mass indicates an angiomyolipoma. F – AML of adrenal is rare++. Myelolipoma can have fat (macroscopic), but can’t differentiate from well differentiated liposarcoma. Perc Bx for Dx.
  7. Both glands are normal in a patient with Cushings disease F – Cushings disease – adrenal hyperplasia due to overproduction of ACTH. 70% of endogenous causes. (CD is in the ACTH dependent group of Cushings syndrome)
57
Q

36.The following statements regarding adrenal imaging are true:

  1. Benign adenomas are typically of high attenuation on CT
  2. Benign adenomas are typically <3cm in diameter on CT.
  3. Cushing syndrome is most commonly caused by a solitary adrenocortical adenoma
  4. Phaeochromocytomas are typically <3cm in diameter
A
  1. Benign adenomas are typically <3cm in diameter on CT. T - <5cm, avg size 2.0-2.5cm.
  2. The following statements regarding adrenal imaging are true: (TW)
  3. Benign adenomas are typically of high attenuation on CT F – UECT <10HU (96% specific), <0 (100% specific)
  4. Benign adenomas are typically <3cm in diameter on CT. T - <5cm, avg size 2.0-2.5cm.
  5. Cushing syndrome is most commonly caused by a solitary adrenocortical adenoma F - Cushing’s disease most common cause (68%). See below.
  6. Phaeochromocytomas are typically <3cm in diameter FCauses of Cushings Syndrome divided into ACTH (corticotrophin) dependent or ACTH independent.ACTH dependent (has bilateral adrenocortical hyperplasia): Cushing’s disease 68%, Ectopic ACTH 12%. (Cushing’s disease = pituitary hypersecretion of ACTH)ACTH-independent CS – adrenal adenoma 10%, Adrenal carcinoma 8%. CME 03.60 (5) (ref. UpToDate)
58
Q

37.With regard to renovascular hypertension:False?

  1. lt accounts for about 5% of all cases of hypertension
  2. Normal pre- and post-captopril scintigraphy excludes renovascular hypertension with a confidence of >90%
  3. Stent insertion is the primary treatment choice in atherosclerotic ostial renal artery stenosis
  4. On Doppler ultrasound,a renal artery to aorta peak systolic ratio of 2.5 in 50%
A
  1. On Doppler ultrasound,a renal artery to aorta peak systolic ratio of 2.5 in 50% F >3.5 for >60% (Dahnert 6th)
  2. With regard to renovascular hypertension: (TW)
  3. lt accounts for about 5% of all cases of hypertension T – 1-5% (Dahnert 6th, eMedicine)
  4. Normal pre- and post-captopril scintigraphy excludes renovascular hypertension with a confidence of >90% T (see below)
  5. Stent insertion is the treatment choice in atherosclerotic ostial renal artery stenosis with resistant hypertension T – Revascularization is usually indicated in pts with hemodynamically significant lesion whith resistant HTN, malignant HTN. Revascularisation procedure of choice perc angio with stent, esp if there is concomitant ostial or proximal renal artery disease. (UpToDate, ACC/AHA 2005 guidelines)
  6. On Doppler ultrasound,a renal artery to aorta peak systolic ratio of 2.5 in 50% F >3.5 for >60% (Dahnert 6th) ACE inhibitor scintigrams should be interpreted as consistent with a low, intermediate, or high probability of renovascular disease.

The most specific diagnostic criterion for RVH at scintigraphy is an ACE inhibitor–induced change on the scintigram….. A normal ACE inhibitor scintigram indicates a low probability (<10%) of RVH. (Radiographics 2000;20:1355)

59
Q
  1. Regarding Phaeochromocytoma, which of the following is false?
  2. They are hyperintense on conventional T2 weighted MRI
  3. There is an association with multiple mucosal neuromas
  4. An extra-adrenal location is associated with a higher likelihood of a malignant histology
  5. bilaterality in 10%
  6. There is an association with hemihypertrophy of the body
  7. There is an association with von-Hippel-Lindau disease
A

5.There is an association with hemihypertrophy of the body F

Bilateral in 10%.
Marked T2 hyperintensity due to cystic areas. 10% occur in extra-adrenal sites and are called paragangliomas. Theses have a higher incidence of malignancy(20-40% vs 10%).
10% of phaeochromocytomas occur in familial syndromes. These include VHL, NF-1, MEN-2A, MEN-2B and Sturge-Weber.
MEN-2B patients have multiple mucosal neuromas or ganglioneuromas at a variety of sites.
Adrenocortical carcinoma, not phaeochromocytoma, is associated with congenital hemihypertrophy.Reference: Robbins 7th ed.; Dahnert 6th ed.

60
Q

39.Regarding adrenocortical adenoma, which of the following is true?

  1. Chemical shift MR is most accurate in differentiating adenoma from metastasis
  2. Attenuation > 10 HU on NECT is typical
  3. Less than 50% washout on 10 minute delayed CECT is typical
  4. Hypointensity to spleen on out-of-phase T1WI is typical
  5. All hyperintensity adenomas are > 2 cm
A

1.Chemical shift MR is most accurate in differentiating adenoma from metastasis

61
Q

40.Which of the following is true regarding blunt renal trauma

  1. Cortical rim nephrogram indicates renal devascularization
  2. Most contusions and lacerations communicate with the collecting system
  3. Renal pedicle injuries are almost always associated with gross hematuria
  4. Gross hematuria almost always indicates serious renal damage
A

1.Cortical rim nephrogram indicates renal devascularization - T
Not certain about this one. “Imaging of Renal Trauma: A Comprehensive Review”, Radiographics, 2001: minor injuries (not involving collecting system acount for up to 85% of blunt renal trauma; haematuria is absent in 24% with thrombosis of the renal artery due to an intimal flap (a form of pedicle injury), and the cortical rim sign is associated with this injury (due to continued perfusion of outermost cortex by the capsular artery, which branches early from the renal artery and is often spared). “MDCT Findings of Renal Trauma”, AJR , 2006: Gross haematuria is the most reliable indicator of serious urologic injury, although the degree of haematuria does not correlate with the degree of renal injury. There are also Pubmed references to gross haematuria corresponding to higher AAST grades of inury.

So, gross haematuria indicates severe injury, but its absence does not exclude severe injury. By this reckoning, the last stem is also correct.

Classification:
Superficial cortical laceration (75-85%)Subcapsular hematomalenticular-shaped area + flattening of subjacent parenchymaRenal contusionpoorly defined area of low attenuation
Small cortical laceration without caliceal disruption

Rx:observationComplete cortical laceration/fracture communicating with caliceal system (10%)extravasation of contrast materialseparation of renal poles (= fracture)Rx:clinical judgement requiredShattered kidney / injury to the renal vascular pedicle (5%)multiple separate renal fragments (= shattered kidney)lack of enhancement of part / all of kidney± “rim sign” (= enhancement of renal periphery through intact capsular / collateral vessels)extravasation of contrast materialRx:surgeryDDx:respiratory motion artifact (low-attenuation area surrounding kidney)

62
Q

41.In renal nuclear medicine which of the following statement is FALSE

  1. Effective renal plasma flow can be measured by Tc 99m DMSA
  2. Effective renal plasma flow can be measured by Tc 99m MAG 3
  3. Glomerular filtration rate can be assessed by Tc 99m DTPA
  4. Tc 99m DMSA imaging is the method of choice in assessing renal scars in reflux nephropathy
  5. Tc 99m MAG 3 is more reliable than Tc 99m DTPA in assessing urinary obstruction in neonates
A

1.Effective renal plasma flow can be measured by Tc 99m DMSA F – cortical scarringDMSA is a cortical agent. Note that glomerular filtration (required for DTPA imaging) does not mature until about 1 month of age.Reference: Mettler, Essentials of Nuclear Medicine Imaging, 5th ed.CME 02.50

63
Q
  1. Which of the following statements regarding neuroblastoma in children is TRUE:
  2. Almost all occur in the abdomen
  3. Peak age of incidence is 5 years
  4. Neuroblastoma in infants has a worse prognosis than those occurring in later childhood
  5. Spread across the midline with vascular encasement is a recognised appearance on ultrasound and CT
  6. Bone scintigraphy with 99mTc-Pi1DP is more accurate than 1 131-MIBG for detection of skeletal mestastases
A
  1. Spread across the midline with vascular encasement is a recognised appearance on ultrasound and CT T48.Which of the following statements regarding neuroblastoma in children is TRUE: (AB)
  2. Almost all occur in the abdomen F – thorax 15%, neck 5%, other 10%
  3. Peak age of incidence is 5 years F – peak age is 2 years
  4. Neuroblastoma in infants has a worse prognosis than those occurring in later childhood F – better prognosis if < 1 year
  5. Spread across the midline with vascular encasement is a recognised appearance on ultrasound and CT T
  6. Bone scintigraphy with 99mTc-Pi1DP is more accurate than 1 131-MIBG for detection of skeletal mestastases F - we use MIBG here

About 60% occur within the abdomen. Neuroblastoma has a perculiar Stage IV-S: age < 1 year; localised primary tumour; dissemination limited to skin, liver, bone marrow. Prognosis approaches almost 100%, often with spontaneous regression. MIBG and bone scans have similar sensitivities (about 70%), but bone scans are less specific. Bone scans have a role in detecting non-MIBG-avid metastases.Refereence: UpToDate

64
Q
  1. The following statements regarding testicular ultrasound are true:
  2. Testicular microlithiasis is associated with Klinefelter syndrome
  3. Cysts are more typically seen in seminomas than yolk sac/ endodermal sinus tumours
  4. Seminomas typically contain cystic elements on US
  5. The majority of teratomas in childhood are malignant
  6. Teratoma is the commonest tumour occurring in association with testicular microlithiasis
A
  1. Testicular microlithiasis is associated with Klinefelter syndrome - T
  2. The following statements regarding testicular ultrasound are true: (AB)
  3. Testicular microlithiasis is associated with Klinefelter syndrome - T
  4. Cysts are more typically seen in seminomas than yolk sac/ endodermal sinus tumours - F
  5. Seminomas typically contain cystic elements on US - F – solid, homogeneous, hypoechoic
  6. The majority of teratomas in childhood are malignant F – change of CME stem
  7. Teratoma is the commonest tumour occurring in association with testicular microlithiasis - F - seminoma

Answer: A is the unambiguously correct answer.Microlithiasis is associated with Klinefelter syndrome. Seminomas tend to be homogeneous at ultrasound, with cysts rare (though Dahnert reports cystic elements in 10%…). Testicular teratomas behave benignly in young children. As for the tumour most commonly associated with microlithiasis, I’m not so sure. Germ cell tumours as a class are, and seminoma is the most common germ cell tumour. Two case studies in Radiology with limited numbers examined this question with conflicting results re: seminoma.

65
Q
  1. In ultrasound of the scrotum, which is true:
  2. Intratesticular lesions are most often malignant.
  3. Seminomas tend to occur in a younger age group than do teratomas.
  4. High resolution grey scale sonography can distinguish between acute epididymitis and torsion.
  5. 90% of patients with acute epididmyitis have an associated orchitis.
  6. It reliably differentiates teratoma from seminoma
  7. decreased resistance if inflammatory changes
A

1.Intratesticular lesions are most often malignant. TTorsion can occasionally cause changes in the epididymis, causing a similar appearance on grey-scale. Orchitis occurs in 20-40% with acute epididymitis. Flow in the inflamed testis is usually increased (low RI) unless swelling within the rigid tunica albuingea compromises flow (high RI). Most intratesticular masses are malignant, particularly palpable solid or complex cystic lesions. Most paratesticular masses are benign.Reference: Dahnert, 6th ed.; eMedicine

66
Q
  1. Medial Displacement of the ureter is seen in
  2. Sacrosciatic hernia
  3. Ureterocoele
  4. ovarian tumor
  5. Prostatomegaly
  6. AP resection
A

5.AP resection T -AP resection is a recognised cause of medial deviation of the distal ureter. An ovarian mass would cause lateral displacement. In ureteric duplication, the upper moiety-associated ureterocoele can cause lateral displacement of the lower moiety ureter.

67
Q

46.The following associations are well recognised:

  1. Escherichia coli infection and malacoplakia
  2. Renal tuberculosis and transitional cell carcinoma
  3. Medullary sponge kidney and renal cell carcinoma
  4. Renal tubular acidosis and radiolucent renal calculi
A

1.Escherichia coli infection and malacoplakia TE. coli infection is well recognised as a cause of malacoplakia. Renal tubular acidosis type 1 causes calcium phosphate (radiopaque) calculi. Only isolated case reports of TB/TCC and medullary sponge/RCC on Pubmed.

68
Q

47.There is a recognised association between all of the following EXCEPT:

  1. Pelvicalyceal obstruction and Xanthogranulomatous pyelonephritis
  2. Calcification in a non-peripheral renal mass and malignancy
  3. Medullary sponge kidney and Caroli’s disease
  4. Wolman’s disease and adrenal atrophy
  5. Schistosomiasis and bladder calcification
A

4.Wolman’s disease and adrenal atrophy - FBilateral extensive calcification in enlarged normally-configured adrenals is diagnostic of Wolman disease.

69
Q

48.With regards to multicystic dysplastic kidney which of the following is FALSE

  1. There is an increased risk of malignancy
  2. There is an increased incidence of pelvi-ureteric junction obstruction in the contralateral kidney
  3. The ureter is atretic
  4. In the hydronephrotic form, a dominant cyst is seen in the pelvis
  5. In the pelvoinfundibular form, the cysts do not communicate
A
  1. There is an increased risk of malignancy F – low risk of malignancy. Life time risk estimates 1:2000 – 1:3000 based on available evidence. Risk of dying from an associated Wilm’s tumour is about 1:20,000 [Weiss, Comprehensive Urology]
  2. With regards to multicystic dysplastic kidney which of the following is FALSE (CC)
  3. There is an increased risk of malignancy F – low risk of malignancy. Life time risk estimates 1:2000 – 1:3000 based on available evidence. Risk of dying from an associated Wilm’s tumour is about 1:20,000 [Weiss, Comprehensive Urology]
  4. There is an increased incidence of pelvi-ureteric junction obstruction in the contralateral kidney T – PUJ obstruction in about 7 – 27% [Dahnert]
  5. The ureter is atretic T - Dahnet suggests that ureteric obstruction/atresia is the underlying cause. Weiss says this may be a consequence of the genetics but I doubt the examiners read Weiss and then again everything is a spectrum - so True. Prerequisites uses this term an ultrasound sign.
  6. In the hydronephrotic form, a dominant cyst is seen in the pelvis T – the dominant cyst is present in the hydronephrotic form. Paediatric Prerequisites.
  7. In the pelvoinfundibular form, the cysts do not communicate T –Pelvoinfundibular subtype of MCDK is the more common. Dahnert quotes 93% accuracy in using non communicating cysts as a diagnostic sign for MCDK.
70
Q

49.Which of the following statements regarding renal cystic disease is FALSE:

  1. Calcification has been reported in 1-3% of renal cysts
  2. The risk of malignancy in a cyst with calcification is about 50%
  3. Hyperdense cysts on CT are a feature of autosomal dominant polycystic kidney disease
  4. If septations in a cyst are smooth and less than 1 mm thick they are of no clinical significance
  5. Cysts develop in up to 50% of patients on long-term dialysis
A

*AJL - I would favour ‘5.Cysts develop in up to 50% of patients on long-term dialysis’ to be the most incorrect. Cysts develop in around 90% of people on long term dialysis.
Answer below is getting a bit complicated.

Previous answer:
2.The risk of malignancy in a cyst with calcification is about 50% - F Type III Bosniak lesion has a 50% chance of being malignant. If the cyst has suspicious calcification with enhancement then this stem is true according to 2002 Bosniak as this makes a renal cyst type III. Hartman goes on to quote Bosniak that qualitative assessment rather than quantitative assessment has not been and probably never will be quantifiable; as some observers will classify IIF as III due to what is perceived to be obscured wall enhancement rather than calcification per se. The stem doesn’t clarify what type of calcification pattern, and note that Dahnert & Hartman put irregular calcification as type III Bosniak which could makes this stem true if the description is “irregular calcification’.

  1. Which of the following statements regarding renal cystic disease is FALSE: (CC)1.Calcification has been reported in 1-3% of renal cysts - T – quote from Radiographics 2004 Hartman Refresher Course Practical Guide to the Cystic Renal Mass, which quoted original article dated 1982.
  2. The risk of malignancy in a cyst with calcification is about 50% - F Type III Bosniak lesion has a 50% chance of being malignant. If the cyst has suspicious calcification with enhancement then this stem is true according to 2002 Bosniak as this makes a renal cyst type III. Hartman goes on to quote Bosniak that qualitative assessment rather than quantitative assessment has not been and probably never will be quantifiable; as some observers will classify IIF as III due to what is perceived to be obscured wall enhancement rather than calcification per se. The stem doesn’t clarify what type of calcification pattern, and note that Dahnert & Hartman put irregular calcification as type III Bosniak which could makes this stem true if the description is “irregular calcification’.
  3. Hyperdense cysts on CT are a feature of autosomal dominant polycystic kidney disease,- T – cyst haemorrhage in 66% [ Dahnert]
  4. If septations in a cyst are smooth and less than 1 mm thick they are of no clinical significance - T - they are non surgical lesions [Radiographics 2004 Hartman Refresher Course Practical Guide to the Cystic Renal Mass].
  5. Cysts develop in up to 50% of patients on long-term dialysis - T After 3 yrs 10- 20 % develop renal cysts After 5 – 10 yrs 90% develop cysts. 7% develop solid renal neoplasm’s – adenoma, adenocarcinoma, oncocytoma [GU Requisites]CME 02.31 (16)
71
Q

50.Which of the following statements is TRUE

  1. An increasing dense nephrogram is a feature of acute glomerulonephritis
  2. A rim nephrogram is seen in acute renal artery occlusion
  3. Renal papillary necrosis is unilateral in about 50% of cases
  4. Hypertension due to renal cell carcinoma does not respond to nephrectomy
  5. Cortical calcification due to cortical necrosis is classically punctate
A
  1. A rim nephrogram is seen in acute renal artery occlusion - T – due to collateral flow from capsular, peripelvic and peri ureteric vessels. Also seen in Renal VV Thrombosis, Acute tubular necrosis, Severe chronic urinary obstruction
  2. Which of the following statements is TRUE: (CC)
  3. An increasing dense nephrogram is a feature of acute glomerulonephritisT – [Dahnert] due to diminished plasma clearance of contrast material
  4. A rim nephrogram is seen in acute renal artery occlusion - T – due to collateral flow from capsular, peripelvic and peri ureteric vessels. Also seen in Renal VV Thrombosis, Acute tubular necrosis, Severe chronic urinary obstruction
  5. Renal papillary necrosis is unilateral in about 50% of cases - F - However, I can’t find a recent incidence on bilateral/unilateral papillary necrosis. eMedicine is all I could find and from an old study (1952), “In general, renal papillary necrosis is more common in women than in men. Mandel organized the first comprehensive review to focus attention on renal papillary necrosis. In his series, which examined 160 cases of renal papillary necrosis from the world literature, 96 patients (60%) had diabetes mellitus, 48 patients (30%) had urinary tract obstruction, and 15 patients (9.4%) had both. In the group with diabetes, women outnumbered men, and in the group of patients without diabetes, men outnumbered women, further reflecting the frequency and significance of urinary tract obstructions in elderly men with renal papillary necrosis. An autopsy report from 1957 by Simon and associates documented the presence of acute or chronic pyelonephritis in 95%.”. Assuming most urinary tract obstruction is unilateral, an incidence of about 30% or less would expected to be unilateral.
  6. Hypertension due to renal cell carcinoma does not respond to nephrectomy – F Hypertension from Renin secreting RCC often normalizes with nephrectomy [Weiss, comprehensive urology, which quotes Sufrin seminars in urology 1989]5.Cortical calcification due to cortical necrosis is classically punctuate - T – [Dahnert] “tramline/punctate calcifications along margins of viable and necrotic tissue as early as 1 -2 months.”
72
Q
  1. In complete duplication of the urinary collecting system which of the following statements is TRUE
  2. Incomplete duplication is associated with pelvi-ureteric obstruction of the upper pole moiety
  3. An associated ectopic ureterocele most often affects the lower pole moiety
  4. The ectopic system may cause wetting in boys
  5. At intravenous urography, an associated ureterocele is best visualised when the bladder is distended
  6. On IVP, downward displacement of the lower pole calyces from obstruction of the non-orthotopic ureter is known as the drooping lily sign
A
  1. On IVP, downward displacement of the lower pole calyces from obstruction of the non-orthotopic ureter is known as the drooping lily sign T
  2. In complete duplication of the urinary collecting system which of the following statements is TRUE (CC)
  3. Incomplete duplication is associated with pelvi-ureteric obstruction of the upper pole moiety F – PUJ occurs at lower pole [Dahnert 6th ed p987-8]
  4. An associated ectopic ureterocele most often affects the lower pole moiety, F – ureterocele = upper pole [Dahnert]
  5. The ectopic system may cause wetting in boys F – no, is above sphincter in boy [Dahnert]
  6. At intravenous urography, an associated ureterocele is best visualised when the bladder is distended F – early filling shot [MCH protocol]
  7. On IVP, downward displacement of the lower pole calyces from obstruction of the non-orthotopic ureter is known as the drooping lily sign T
73
Q

52.The following statements are true regarding complications after renal transplantation

  1. Lymphocoeles accumulate radiotracer
  2. Lymphocoeles appear in the first 4 days following transplantation
  3. Renal artery Doppler velocity greater than 1m/sec indicates renal artery stenosis
  4. The combination of absent venous flow and reversal of arterial waveform is characteristic of renal vein thrombosis
  5. Normal ultrasound examination excludes rejection.
A
  1. The combination of absent venous flow and reversal of arterial waveform is characteristic of renal vein thrombosis - TT - Four direct criteria are used for proximal stenosis or occlusion of a renal artery: An increase in the peak systolic velocity in the renal artery > 150 cm/s for angles < 60 degree or >180 cm/s for angles > 70 degrees (in the literature, the threshold for significant RAS is 100 – 200 cm/s). A renal to aortic ratio of peak systolic velocity more than 3.5, Turbulent flow and/or flow reversal in the post stenotic area, Visualization of the renal artery without detectable Doppler signal, a finding that indicates occlusion. Absent venous flow and u-shaped/plateau-like reversal of diastolic arterial flow (renal vein thrombosis).
  2. The following statements are true regarding complications after renal transplantation (CC)
  3. Lymphocoeles accumulate radiotracer - F – photopaenic defect from displacement/impression on the renal transplant/urinary bladder [Dahnert 6th ed p 964]
  4. Lymphocoeles appear in the first 4 days following transplantation – F [Dahnert 6th ed p 964] Lymhocoeles form within 4 – 8 weeks after transplantation.
  5. Renal artery Doppler velocity greater than 1m/sec indicates renal artery stenosis – F - >180-200cm/sec depending on what you read. 2:1 ratio of peak stenotic and post stenotic, dampened signal post
  6. The combination of absent venous flow and reversal of arterial waveform is characteristic of renal vein thrombosis - TT - Four direct criteria are used for proximal stenosis or occlusion of a renal artery: An increase in the peak systolic velocity in the renal artery > 150 cm/s for angles < 60 degree or >180 cm/s for angles > 70 degrees (in the literature, the threshold for significant RAS is 100 – 200 cm/s). A renal to aortic ratio of peak systolic velocity more than 3.5, Turbulent flow and/or flow reversal in the post stenotic area, Visualization of the renal artery without detectable Doppler signal, a finding that indicates occlusion. Absent venous flow and u-shaped/plateau-like reversal of diastolic arterial flow (renal vein thrombosis).
  7. Normal ultrasound examination excludes rejection. - F – F in acute rejection ultrasound has a 30 – 50 % negative predictive value only, signs are [Dahnert]: Increase in renal volume from oedema, including decreased renal sinus fat, increased cortical thickness, Conspicuous pyramids and reduced cortical echogenicity, Thickening of the pelvoinfundibular wall, Diminished echogenicity of fat
74
Q

53.The following are features of xanthogranulomatous pyelonephritis:

  1. The kidney is typically small
  2. Focal involvement is more common than diffuse involvement
  3. Is associated with non functioning kidney in 75%
  4. The majority of cases are bilateral
  5. Calculi are present in 10%
A
  1. Is associated with non functioning kidney in 75% - T – non functioning enlarged kidney is typical (80% have absent nephrograms) [Dahnert] although variable rim enhancement surrounding the central inflammatory process is common [Radiographics 2008, Craig]
  2. The following are features of xanthogranulomatous pyelonephritis: (CC)
  3. The kidney is typically small - F – Kidney is globally enlarged (smooth contour uncommon) [Dahnert]. Granulomatous inflammatory process (replaced by foamy macrophages) from chronic obstruction tetrad of :Non functioning enlarged kidney,Central calculus with contracted renal pelvis,Expansion of the calyces,Perinephric fatty stranding
  4. Focal involvement is more common than diffuse involvement - F – F - Diffuse involvement more common(83 – 90%) [Dahnert]
  5. Is associated with non functioning kidney in 75% - T – non functioning enlarged kidney is typical (80% have absent nephrograms) [Dahnert] although variable rim enhancement surrounding the central inflammatory process is common [Radiographics 2008, Craig]
  6. The majority of cases are bilateral – F –F – most causes are unilateral. Only reference I could find to bilateral XGP isfrom eMedicine quoting Perez, J Urol, 1993. It states that bilateral XGP is usually fatal and that bilateral form can be treated with bilateral partial nephrectomy.
  7. Calculi are present in 10%F – Calculi are absent in 10%, present ~80%.
75
Q

54.Of the following statements which is FALSE

  1. Central calcification within a renal mass is highly suggestive of malignancy
  2. Milk of calcium pattern of stone formation occurs proximal to a pelviureteric junction obstruction
  3. Cystine stones are opaque on CT
  4. Indinovir calculi are readily identified as radio-opaque on CT
  5. Cortical nephrocalcinosis is associated with chronic rejection of a transplant kidney
A
  1. Indinovir calculi are readily identified as radio-opaque on CT - Uric acid, xanthine, Matrix (mucoprotein/mucopolysaccharide), Indinavir (antiviral) drug precipitates in renal tubules, are non opaque.
  2. Of the following statements which is FALSE (erics notes) (CC)
  3. Central calcification within a renal mass is highly suggestive of malignancy - T – 15 – 20 % have calcification that is central and amorphous. In cystic type RCC can be curvilinear in nature. [Dahnert ,Warren and Mcfarlane BJU 2004]
  4. Milk of calcium pattern of stone formation occurs proximal to a pelviureteric junction obstruction - T – T – Milk of Calcium can be seen in renal cyst, caliceal diverticulum and obstructed hydrocalyx {Dahnert 6th ed.]
  5. Cystine stones are opaque on CT - T – cystine stones are mildly opaque [Dahnert ]
  6. Indinovir calculi are readily identified as radio-opaque on CT - Uric acid, xanthine, Matrix (mucoprotein/mucopolysaccharide), Indinavir (antiviral) drug precipitates in renal tubules, are non opaque.
  7. Cortical nephrocalcinosis is associated with chronic rejection of a transplant kidney – T – rejected renal transplant is a cause [Dahnert 6th ed.]Cortical nephrocalcinosis:Acute cortical necrosisChronic GNAlport syndrome = hereditary nephritis and deafnessCongenital oxalosisChronic paraneoplastic hypercalcaemiaToxic : ethylene glycol, methoxyfluraneSCARejected renal transplant
76
Q
  1. In doppler examination of the kidney the following are true statements T/F
  2. A high resistive index indicates a non-obstructive cause of acute renal failure
  3. Renal to aortic peak systolic velocity ratio greater than 1.5 = renal artery stenosis
  4. Increase in the resistive index in the interlobar arteries = renal artery stenosis
  5. The RI measurement is independent of heart rate
  6. The RI is dependent on the angle of insonation
  7. Mag 3 ….
A

all false

  1. In doppler examination of the kidney the following are true statements T/F
  2. A high resistive index indicates a non-obstructive cause of acute renal failure F – RI does inc in acute obstruction but is often (N) in chronic obstruction. RI is raised in a large percentage of kidneys with acute, high grade obstruction
  3. Renal to aortic peak systolic velocity ratio greater than 1.5 = renal artery stenosis F - Four direct criteria are used for proximal stenosis or occlusion of a renal artery:,An increase in the peak systolic velocity in the renal artery > 150 cm/s for angles < 60 degree or >180 cm/s for angles > 70 degrees (in the literature, the threshold for significant RAS is 100 – 200 cm/s), A renal to aortic ratio of peak systolic velocity more than 3.5, Turbulent flow and/or flow reversal in the post stenotic area,Visualization of the renal artery without detectable Doppler signal, a finding that indicates occlusion.
  4. Increase in the resistive index in the interlobar arteries = renal artery stenosis F – loss of early systolic peak in interlobar arteries, so reduced RI.
  5. The RI measurement is independent of heart rate F - RI depend on heart rate. Corrected RI = observed RI – 0.0026(80 – observed heart rate) [ Mostbeck, Radiology 1990]
  6. The RI is dependent on the angle of insonation F - angle of insonation effects absolute velocity measurements, RI and SD ratios cancel these errors out [ McGahan, diagnostic ultrasound ]
  7. Mag 3 …. (incomplete)
77
Q

56.The following statements regarding imaging of the adrenal adenomas are true

  1. The presence of calcification in an adrenal mass effectively excludes carcinoma
  2. They are hyperintense to liver on T2 weighted MR scans
  3. The average size at presentation is less than 3cm.
  4. In patients with known lung cancer, an adrenal lesion is likely to be metastatic
  5. Low signal on out of phase MR implies malignancy
A
  1. The average size at presentation is less than 3cm. T Dahnert: average size 2.0-2.5cm.
  2. The following statements regarding imaging of the adrenals adenomas are true (CC, TW)
  3. The presence of calcification in an adrenal mass effectively excludes carcinoma F – Malignant Calcification can be found in [Dahnert]:Neuroblastoma, Phaechromocytoma, Adrenal carcinoma, Metastases
  4. They are hyperintense to liver on T2 weighted MR scans F (AJR 1986: most of the adrenal adenomas displayed low T2 signal with adrenal/liver ratio of 1.2. Phaeo adrenal / liver ratio was >3.4). T - [Dahnert] iso/hypointense on T1 to spleen. If hypertense to liver suspect carcinoma . Phaeos are hyperintense to spleen on T2. But T2WI cannot be relied upon to determine adenoma from non adenoma when compared with the liver [baker, AJR 1989]
  5. The average size at presentation is less than 3cm. T Dahnert: average size 2.0-2.5cm.
  6. In patients with known lung cancer, an adrenal lesion is likely to be metastatic F
    * AJL disagrees. Lung mets to adrenal commonly therefore that is the first thing to exclude.

5.Low signal on out of phase MR implies malignancy F Adenoma has fat and water signals which cancel out by > 20% on opposed phase sequences [Dahnert 6th ed. P919]Added word ‘adenomas’ to question. Added option ‘d’ and ‘e’.

78
Q

57.Regarding CT of the adrenals, the following statements which is FALSE:

  1. Aldosteronomas are typically hypodense
  2. In Cushing’s syndrome due to hyperplasia, the adrenals are enlarged but usually normal in configuration
  3. Phaeochromocytomas are bilateral in approximately 10% of cases
  4. Phaeochromocytomas are typically < 2cm in size at presentation
  5. Adrenal carcinoma is found in about 5% of patients with Cushing’s Syndrome
A

4.Phaeochromocytomas are typically < 2cm in size at presentation F – average 5 cms and range from 3 – 12 cms [Dahnert 6th ed]6

  1. Regarding CT of the adrenals, the following statements which is FALSE: (CC)
  2. Aldosteronomas are typically hypodense T – referring to adrenal adenoma which are typically hypodense (<10HU on non-con)
  3. In Cushing’s syndrome due to hyperplasia, the adrenals are enlarged but usually normal in configuration T – [Dahnert], p925 6th ed.] Note the insertion of the phrase “due to hyperplasia” which makes this stem true, without which the stem could be false as adrenal adenomas can cause hypercortisolism and adrenals can be normal in up to 50% of patients with Cushing’s disease ( overproduction of pituititary ACTH). [Primer 4th ed]
  4. Phaeochromocytomas are bilateral in approximately 10% of cases T- [Dahnert p 944 6th ed.] – the 10 % tumour
  5. Phaeochromocytomas are typically < 2cm in size at presentation F – average 5 cms and range from 3 – 12 cms [Dahnert 6th ed]
  6. Adrenal carcinoma is found in about 5% of patients with Cushing’s Syndrome T – in Adults about 5- 10 %, in children about 66% [Dahnert 6th ed. P 925]
79
Q

58.Retoperitoneal Teratomas, which of the following is false?

  1. A cystic teratoma is likely to be malignant where as a solid teratoma is likely to be benign.
  2. Benign teratomas are bilateral in 10 – 15%
  3. immature malignant teratomas are chiefly found in prepubertal females
  4. Monodermal teratomas can present with hyperthyroidism
  5. pseudohermaphratism is associated with dysgerminomas
A

1.A cystic teratoma is likely to be malignant where as a solid teratoma is likely to be benign. F - A cystic teratoma is likely to benign where as a solid teratoma is likely to be malignant. [Robbins]

  1. Retoperitoneal Teratomas, which of the following is false? (CC)
  2. A cystic teratoma is likely to be malignant where as a solid teratoma is likely to be benign. F - A cystic teratoma is likely to benign where as a solid teratoma is likely to be malignant. [Robbins]
  3. Benign teratomas are bilateral in 10 – 15% T - Benign teratomas are bilateral in 10 – 15%. [Robbins]
  4. immature malignant teratomas are chiefly found in prepubertal femalesT - immature malignant teratomas are chiefly found in prepubertal females [Robbins]
  5. Monodermal teratomas can present with hyperthyroidism T – Monodermal or specialized teratomas include strumii ovarii, ovarian carcinoid can present with paraneoplastic symptoms from secreting thyroid hormone and serotonin [Robbins]
  6. pseudohermaphratism is associated with dysgerminomas T - Dysgerminomas occur in gonadal dysgenesis syndromes including pseudohermaphratism is Initial question read “retroperitoneal teratomas”… that was it, no options. So wrote question
80
Q

59.Malakoplakia and E Coli which of the following statements is incorrect?

  1. The most frequent renal finding is obstruction secondary to a lesion in the lower tract
  2. Uncommon inflammatory response to gram positive organisms
  3. Surgery and taking vitamin C are treatments
  4. Calcium deposition in Intracellular inclusion bodies is a characteristic of renal malakaplakia
  5. Gallium scintigraphy can detect clinically silent lesions
  6. Nearly 50% of patients have serious systemic disease or are immunosuppressed
A

LW: Preferred answer that is incorrect is 3: Surgery and taking Vitamin C are treatments.

Malakoplakia is an inflammatory condition due to gram negative bacteria, vast majority is E.Coli, (hence uncommon to be due to gram positive organisms).
Involves a defect in the phagolysosome mechanism of digestion resulting in Michaelis Gutmann bodies which contain Ca++ due to bacterial digestion.
These lesions can cause obstruction of the lower urinary tract.
Mainstay of treatment is ABx targeting E.Coli and that have a high concentration within the macrophages. Cannot find Vitamin C mentioned as main stay treatment. Surgery can often be required. Thus why Vitamin C option is incorrect.
Patients commonly have a systemic disease / immune suppressed.
* As radiologists, who dont actually treat patients, I think it is beyond the scope of this exam to know rare / uncommon treatment options*.

Prior notes:
2.Uncommon inflammatory response to gram positive organisms F – E. coli in 94% [Dahnert]65.Malakoplakia and E Coli which of the following statements is incorrect? (CC)1.The most frequent renal finding is obstruction secondary to a lesion in the lower tract T - Because malacoplakia most commonly manifests as a mucosal mass involving the bladder or ureter, the most frequent renal finding is obstruction secondary to a lesion in the lower tract [Radiographics 2008 Pyelonephritis:Rad-path review]2.Uncommon inflammatory response to gram positive organisms F – E. coli in 94% [Dahnert]3.Surgery and taking vitamin C are treatments 4.Calcium deposition in Intracellular inclusion bodies is a characteristic of renal malakaplakia T – Calcum deposition in Michaelis – Gutmann bodies are a characteristic histological finding of renal malakoplakia from phagocytosied bacterial elements. [Radiographics 2008 Pyelonephritis:Rad-path review]5.Gallium scintigraphy can detect clinically silent lesionsT - Gallium scintigraphy can detect clinically silent lesions [Weiss] 6.Nearly 50% of patients have serious systemic disease or are immunosuppressed T - Nearly 50% of patients have serious systemic disease or are immunosuppressed. Associated mortality in 15%, worse prognosis if upper tracts involved. [Weiss]uncommon chronic inflammatory response to Gram-negative infectionOrganism:E. coli (in 94%);diabetes mellitus predisposesHisto:submucosal histiocytic granulomas containing large foamy mononuclear cells (Hansemann macrophages) withintracytoplasmic basophilic PAS-positive inclusion bodies (Michaelis-Gutmann bodies) consisting of incompletelydestroyed E. coli bacterium surrounded by lipoprotein membranesPeak age:5th-7th decade; M:F = 1:4hematuriaraised yellow lesion <3 cm in diameterLocation:bladder > lower 2/3 of ureter > upper ureter > renal pelvis; multifocal in 75%; bilateral in 50%multiple dome-shaped smooth mural filling defectsscalloped appearance if lesions confluentgeneralized pelviureteral dilatation (if obstructive)displacement of pelvicaliceal system + distorted central sinus complexmultifocal parenchymal masses may cause diminished / absent nephrogramDDx:pyeloureteritis cystica

81
Q
  1. Prostate Cancer. which one is false
  2. Has a high signal on T2 weighted images
  3. On MR spectroscopy has an increase in the choline peak and decrease in citrate peak.
  4. Arises in the peripheral zone
  5. Is hyper echoic on ultrasound
A
  1. Has a high signal on T2 weighted images F – low signal in the normally high signal peripheral zone
  2. Prostate Cancer
  3. Has a high signal on T2 weighted images F – low signal in the normally high signal peripheral zone
  4. On MR spectroscopy has an increase in the choline peak and decrease in citrate peak. (Choline rises/citrate declines with increasing malignancy)- cancer: inc choline, dec citrate
  5. Arises in the peripheral zone T – 70% PZ > TZ > CZ
  6. Is hyper echoic on ultrasound T -*AJL (30% of the time) hypoechoic 70% of the time.
82
Q

61.The plain film changes of a right perirenal abscess include: which is false

  1. Anterior displacement of the descending duodenum
  2. Loss of the properitoneal fat strip
  3. Non obliteration of the right psoas shadow
  4. The inferior angle of the liver is obscure
  5. A scoliosis concave to the left
A
  • LW:
    1. Anterior displacement of the descending duodenum: TRUE
    2. Loss of the properitoneal fat strip: TRUE
    3. Non obliteration of the right psoas shadow: FALSE, as you loose the psoas shadow, i.e. it is obliterated.
    4. The inferior angle of the liver is obscure: TRUE
    5. A scoliosis concave to the left: TRUE
  1. Anterior displacement of the descending duodenum - T
  2. Loss of the properitoneal fat strip - T

Plain abdominal film findings loss of renal outline, loss of psoas shadow, scoliosis convex to the side of infection, and immobilization of the kidney on respiration on urographic studies.
Gas is seen in fewer than 20% of patients, the majority of whom are diabetic (see section on emphysematous pyelonephritis).

Chest radiographs may show pleural fluid, lower-lobe air space disease, or elevation of the hemidiaphragm on the affected side

The most common site for collection of pus is behind the kidney in the posterior or dorsal perinephric space

Diabetes mellitus is present in 30% of patients. Pyuria is present and urine cultures are positive in 60% of patients.

83
Q
  1. Regarding Testicular Ultrasound
  2. An intra testicular varicocoele is always associated with an extra testicular varicocoele
  3. An intra testicular spermatocoele can be differentiated from a testicular cyst
  4. An epidermoid of the testis has a characteristic onion skin appearance
  5. 95% of intra testicular masses are malignant
  6. The majority of lesions detected with scrotal ultrasound are intra testicular compared to extra testicular
A
  1. An epidermoid of the testis has a characteristic onion skin appearance T
  2. 95% of intra testicular masses are malignant T – 90-95% of testicular tumours are malignant
  3. Regarding Testicular Ultrasound
  4. An intra testicular varicocoele is always associated with an extra testicular varicocoele F – can occur in association with extratesticular varicocele, but their independent existence is more common
  5. An intra testicular spermatocoele can be differentiated from a testicular cyst F
  6. An epidermoid of the testis has a characteristic onion skin appearance T
  7. 95% of intra testicular masses are malignant T – 90-95% of testicular tumours are malignant
  8. The majority of lesions detected with scrotal ultrasound are intra testicular compared to extra testicular FEpidermoid cyst rare but benign lesion, accounts for 1-2% of all testicular lesions; higher incidence (R) testes; on US bulls eye/target or onion ring appearance (unique onion ring appearance)
84
Q

63.Prune Belly Syndrome is associated with

  1. Abdominal musculature deficiency
  2. Vesico ureteric reflux
  3. Ureteric obstruction
  4. Posterior urethral stricture
  5. Cryptorchidism
A

*AJL: I think there are multiple correct answers; abdominal musculature deficiency, VUR, cryptorchidism.

LJS edit: no posterior urethral stricture - posterior urethra is dilated but without mechanical obstruction (stat dx)*

*LW: agrees with above.
Favoured answer is cryptochordism.

3.Ureteric obstruction F – non obstructed, dilated ureters. Ureters are dilated and tortuous and have a lack of muscle Typically a male with thin or lax abdominal wall and a long, dilated prostate urethra from prostate hyperplasia. Some have utricle diverticulum from the urethra, tortuous and dilated uteters all have cryptoorchidism

85
Q

64.Medial Displacement of the Ureter is seen in

  1. Pelvic lymphadenopathy
  2. Sacrosciatic hernia
  3. Ureterocoele
  4. Prostatomegaly
  5. AP resection
A

*AJL - I would favour AP resection being most correct.

Previous answer:
1.Pelvic lymphadenopathy 5.AP resection

86
Q

65.Xanthgranulomatous pyelonephritis. which are true?

  1. Is associated with calculi in 20%
  2. Is associated with non functioning kidney in 75%
  3. Is associated with Pseudomonas infection in 70%
  4. Diabetes mellitis is the most common systemic association
  5. Is associated with small kidneys
A
  1. Is associated with non functioning kidney in 75% T

4. Diabetes mellitis is the most common systemic association T

87
Q
  1. Pyelitits cystica is associated with
  2. Infection
  3. Tumour
  4. Diabetes
  5. Stricture
A
  1. Infection T

3. Diabetes T

88
Q

67.
Wilms Tumour is associated with

  1. Beckwith Weideman syndrome
  2. Aniridia
  3. Cerebral gigantism
  4. Vesico ureteric reflux
  5. Hypertrophic pyloric stenosis
A
  1. Wilms Tumour is associated with
  2. Beckwith Weideman syndrome T
  3. Aniridia T
  4. Cerebral gigantism T – Sotos syndrome, 2-3% risk Wilms.
  5. Vesico ureteric reflux ?F
  6. Hypertrophic pyloric stenosis F

Wilms Tumor associated with: WAGR (Aniridia, GU abnormalities, Mental retardation, Wilms); Perlman (AR overgrowth syndrome, fetal gigantism, visceromegaly, unusual face, bilateral renal hamartomas, nephroblastomatosis, wilms)

89
Q

68.Phaeochromocytoma. which one is false

  1. 30% are malignant in children
  2. Adrenal masses are more likely to be malignant than extra adrenal masses
  3. Are associated with mucosal neuromas
  4. Are hyperintense on T2 weighted images
A
  1. Adrenal masses are more likely to be malignant than extra adrenal masses - F
    - -> Extra adrenal pheo’s more commonly malignant
  2. Phaeochromocytoma
  3. 30% are malignant in children - T 30% are extraadrenal in children and those are more likely to be malignant
  4. Adrenal masses are more likely to be malignant than extra adrenal masses - F
  5. Are associated with mucosal neuromas T
  6. Are hyperintense on T2 weighted images T

10 % phaeochromocytomas occur in children, and kids have increased rate of : bilateral, malignant, multifocal, extraadrenal.

90
Q
  1. Regarding Hyperoxaluria. which is false
  2. Usually presents at 3-5 yrs
  3. Associated with osteomalacia
  4. Small hypo dense kidneys
A

3.Small hypo dense kidneys – F hyperdense
(LW: normal sized kidneys also).

  1. Regarding Hyperoxaluria
  2. Usually presents at 3-5 yrs – T (hyperoxaluria is a rare autosomal recessive inherited enzyme deficiency. Primary (hereditary) is more common. Secondary (acquired) is rare (enteric: disturbance of bile-acid metabolism after jejunoileal bypass, ileal resection, and blind loop syndrome))
  3. Associated with osteomalacia – T deposits in bone.
  4. Small hypo dense kidneys – F hyperdense and normal size.
91
Q

71.Regarding Testicular USS. which is false

  1. Microlithiasis in Klinefelters
  2. Mature teratomas in child usually benign
  3. Increased cystic change in embryonal cancer
  4. If epididymis/spermatic cord involved most likely lymphoma
A

4.If epididymis/spermatic cord involved most likely lymphoma F – I think that this is probably true. This may actually be the answer.
Of tumours it is true but in general a lesion involving testes and epididymus is more likely inflammatory (Tanya)

92
Q
  1. Regarding Renal artery stenosis, which is least correct?
  2. Peak velocity of 100cm/sec = normal
  3. In renal artery stenosis the Aortic PSV is less than the renal artery PSV
  4. There is increased RI in subcapsular haematoma
  5. Increased RI excludes obstruction
A
  1. Regarding Renal artery stenosis:
  2. Peak velocity of 100cm/sec = normal T – peak systolic >150cm/sec; ratio of peak renal > peak aortic 3.5
  3. In renal artery stenosis the Aortic PSV is less than the renal artery PSV Y (Aorta is normally 100-150cm/sec. So there would be increased renal artery/aorta PSV ratio > 3)
  4. There is increased RI in subcapsular haematoma T
  5. Increased RI excludes obstruction F
93
Q
  1. Regarding Nuclear renal imaging: which is false
  2. DMSA is for scarring
  3. DTPA is for split renal function
  4. MAG 3 accumulates tracer in ATN
  5. MAG 3 excreted by glomerular filtration
A

4.MAG 3 excreted by glomerular filtration F – Tubular

  1. Regarding Nuclear renal imaging:
  2. DMSA is for scarring T – DMSA: proximal and distal tubular uptake
  3. DTPA is for split renal function T – DTPA: nearly 100% filtered GFR
  4. MAG 3 accumulates tracer in ATN T – MAG3: 99% secreted
  5. MAG 3 excreted by glomerular filtration F – Tubular
94
Q
  1. Regarding Renal transplant:
  2. Lymphocele accumulates tracer
  3. Urinoma accumulates tracer
A
  1. Regarding Renal transplant:
  2. Lymphocele accumulates tracer - F – occurs in 10%, occurs week to months after transplantation, no creatinine.
    * LW: Nuk Med Paper states: “ A lymphocele presents as a persistent photopenic area, although a mild degree of filling-in on delayed imaging has been observed”.

2.Urinoma accumulates tracer - T – does have creatinine.

95
Q
  1. At 24 hours imaging Gallium uptake may be seen in.
  2. Blood pool.
  3. Nasopharynx.
  4. Cervical nodes.
  5. The testis.
  6. The kidney.
A

2.Nasopharynx – T – nasal mucosa
5.The kidney – T
Blood pool - T.

Gallium at 24h: reticulendothelial system, liver, spleen, bone marrow, bowel wall, renal cortex, nasal mucosa, blood pool. Gallium at 72h: equallydistributed among soft tissues. Kidney activity no longer detectable.

96
Q
  1. Regarding ultrasound of the testes.
  2. The epididymis shows decreased echogenicity compared with the testis.
  3. Most intratesticular masses are malignant.
  4. Seminoma occurs in older males than teratoma
  5. 90% of epididymitis is associated with orchitis.
A
  1. Most intratesticular masses are malignant - T
  2. Seminoma occurs in older males than teratoma – T – teratoma = 5yo in 75%.
  3. Regarding ultrasound of the testes.
  4. The epididymis shows decreased echogenicity compared with the testis – F – isoechoic, hetereogeneous.
  5. Most intratesticular masses are malignant - T
  6. Seminoma occurs in older males than teratoma – T – teratoma = 5yo in 75%. Seminoma mean age is 40.5y
  7. 90% of epididymitis is associated with orchitis – F – 20-40%
97
Q
  1. Renal transplant ultrasound. which is false
  2. Doppler ultrasound can reliably distinguish ATN from rejection.
  3. Dilatation of the collecting system without obstruction is common in a normal transplant.
  4. An increase in renal size may be the first sign of rejection.
  5. Vascular complications occur in 15%.
A

1.Doppler ultrasound can reliably distinguish ATN from rejection. F

  1. Renal transplant ultrasound - which is False:
  2. Doppler ultrasound can reliably distinguish ATN from rejection. F
  3. Dilatation of the collecting system without obstruction is common in a normal transplant. T
  4. An increase in renal size may be the first sign of rejection. T – increased renal volume from oedema
  5. Vascular complications occur in 15%. ?T 10% (but radiographics says 1-2%)
98
Q

79.Non-ionic contrast media.which is false

  1. Produce a better nephrogram than ionic contrast media.
  2. Have anti-coagulant effect but is less than with ionic contrast media.
  3. Produce less vaso-dilation than ionic contrast media.
  4. Include hexa-iodinated dimers.
A

1.Produce a better nephrogram than ionic contrast media. FIonic > nonionic with: widespread vasodilatation / fluid overload / direct effects on myocardium / antiplatelet and anticoagulant effects / stimulation of vasopressin (renal).

99
Q

80.Renal imaging. which is false

  1. Leukaemia produces bright enlarged kidneys.
  2. Lymphoma produces multifocal areas of low attenuation.
  3. Infantile polycystic kidney disease shows multiple small cysts on ultrasound.
A
  1. Infantile polycystic kidney disease shows multiple small cysts on ultrasound. - F – usually large echogenic centrally with sonolucent rim compressed cortex (cysts = small < 2mm produce increased echoes). Occasionally see macroscopic cysts <1cm (like MSK – in older children – juvenile)
  2. Renal imaging.
  3. Leukaemia produces bright enlarged kidneys - T
  4. Lymphoma produces multifocal areas of low attenuation. - T - Metastatic or invasion of kidney àdiffuse kidney enlargement, multiple bilateral masses, solitary mass or sinus invasion
  5. Infantile polycystic kidney disease shows multiple small cysts on ultrasound. - F – usually large echogenic centrally with sonolucent rim compressed cortex (cysts = small < 2mm produce increased echoes). Occasionally see macroscopic cysts <1cm (like MSK – in older children – juvenile)
100
Q

83.Xantho-graulomatous pyelonephritis. which one is false?

  1. Extension into the soft tissues is hypoechoic on ultrasound.
  2. Infiltration of perirenal fat is demonstrated on C.T.
  3. A staghorn calculus is often present.
  4. More commonly affects females.
  5. Shows rim enhancement on C.T.
A

HAHAHA THEY ARE ALL TRUE!!!

101
Q

84.Calcification occurs in more than 10% of. (multiple true answers)

  1. Wilm’s tumours.
  2. Neuroblastoma.
  3. Ependymoma.
  4. Endodermal sinus tumours.
  5. Rhabdomyosarcoma.
A
  1. Calcification occurs in more than 10% of.
  2. Wilm’s tumours.?T 10-15%
  3. Neuroblastoma. T
  4. Ependymoma. T
  5. Endodermal sinus tumours. F
  6. Rhabdomyosarcoma. F
102
Q
  1. Vesico-ureteric reflux has the following associations. (multiple true answers)
  2. Sibs with VUR.
  3. Nocturnal enuresis.
  4. Ipsilateral PUJ.
  5. Diverticula.
  6. Duplex collecting system.
A
  1. Vesico-ureteric reflux has the following associations.
  2. Sibs with VUR. T
  3. Nocturnal enuresis. F
  4. Ipsilateral PUJ. F
    * *LJS - true, 30-50% PUJ have contralateral renal abn e.g. VUR
  5. Diverticula. T – hutch diverticulum, ureterocele, bladder outlet obstruction, voiding dysfunction
  6. Duplex collecting system. T
103
Q

86.In CT of the adrenals. which is true:

  1. Approximately 70% of cases of endogenous Cushing’s syndrome are caused by bilateral adrenal hyperplasia
  2. Abundant perinephric fat clearly outlines the adrenals in most patients with Cushing’s syndrome
  3. I.V. contrast is often needed because the tumours are often not well seen.
  4. Phaeochromocytomas found in the MEN II syndrome are usually extra adrenal
  5. I.V. contrast should not be used in patients with suspected phaeochromocytomas
A

LW:
1. Approx. 70% of cases of endogenous Cushings syndrome caused by bilateral adrenal hyperplasia.
Favoured answer: adrenal hyperplasia accounts for 75-80% of Cushing syndrome cases, i.e. Cushings disease with production of ACTH causing bilateral adrenal hyperplasia. (StatDx).

**LJS - most common cause of endogenous Cushings is pituitary microadenoma secreting ACTH (70%). Robbins 9th ed p. 1123

  1. No mention of fat on imaging appearances, however difficult to ascertain what abundant infers, as if there is moderate increase in adrenal size with hyperplasia, there may be relatively reduced peri nephric fat volume. COnversely, common cause of Cushing’s is exogenous steroids which causes adrenal hypoplasia, and hence likely relative increase in surrounding fat….. (I should check this tho).
  2. False: Theoretical risk of induction of hyperadrenergic symptoms with iodinated contrast administration discounted by retrospective series where nonionic material utilized (StatDx).

Prior notes:
5.I.V. contrast should not be used in patients with suspected phaeochromocytomas. T – but in current practice this would be false.

  1. In CT of the adrenals
  2. Approximately 70% of cases of endogenous Cushing’s syndrome are caused by bilateral adrenal hyperplasia F
  3. Abundant perinephric fat clearly outlines the adrenals in most patients with Cushing’s syndrome T
  4. I.V. contrast is often needed because the tumours are often not well seen. F
  5. Phaeochromocytomas found in the MEN II syndrome are usually extra adrenal. F
  6. I.V. contrast should not be used in patients with suspected phaeochromocytomas. T – but in current practice this would be false.
104
Q
  1. The mode of excretion of intravenous contrast agents is
  2. Tubular secretion
  3. Tubular excretion
  4. Tubular secretion and excretion
  5. Glomerular filtration
  6. None of the above
A

4.Glomerular filtration T – 99% filtered

105
Q

88.Ultrasound changes in the “normal” transplant kidney ie. no clinical or biochemical evidence of rejection, ATN or cyclosporin toxicity, include

  1. Prominent sinus echoes
  2. Perinephric fluid collection 14 days post transplant
  3. Full calyceal system
  4. Prominent renal pyramids
  5. increase in renal volume
A
  1. Full calyceal system - T
    * AJL - agree with above. This is because it is plumbed into the top of the bladder therefore calyces fill up (according to Rex)